Sie sind auf Seite 1von 318

1

Training Objective
Static Electricity and Conduction
Electrostatic Law of Attraction and Repulsion
Unit of Charge, Coulomb Law
Conduction of electricity is solid, liquids, gases and vacum


Can be defined MATTER
as anything that has mass
(weight) and occupies space
Weight and Dimension of Matter can be
measured
Matter may be found in any one of three
states : Solid , Liquid & Gaseous,
All matter is made from atoms.

Oxygen atom

Hydrogen atom
4

STATES OF
MATTER

Structure of the
Atom

An atom is the smallest particle of an element possessing the


unique characteristics of that element. Atoms have a planetary
type of structure.
They consist of a central nucleus surrounded by orbiting
electrons.
The nucleus consists of positively charged Protons and
uncharged particles called Neutrons.

Subatomic
There are three smaller particles that make up
individual atoms, these are called subatomic .The
three particles are different in size and charge.
Neutrons have no charge and are the largest
subatomic particles. Protons are slightly smaller than
neutrons and have a positive (+) charge. Electrons are
the smallest of all and carry a negative (-) charge

Balanced or Neutral charge


Neutrons & Proton are held together in the Nucleus. The
electrons orbit the nucleus at great speed and distance. Their
speed prevents them from collapsing into the nucleus. An atom
has the same number of electrons and protons, the atom is
balanced or neutral charge

Ionization is the
gains electron
Neutral Atom

process
by which an atom losses or
Ionization
Negative Ion

Positive Ion

The Atom losing an Electron becomes a


Positive ion
The Atom gaining an Electron becomes a
Negative ion

Ionization Energy
Ionization energy is the energy required by an atom there
must be transfer of energy which results in a change in
the internal energy of the atom to remove one electron to
form Positive ions or Cattion , an electron released an atom to
form Negative ions , or Anion

10

Definition of "ionization"
The process of ionizing; the formation of
ions by separating atoms or molecules by
adding or subtracting electrons from
atoms by :
Strong electric fields in a gas
Heat
Radiation
Chemical reaction
Electrical discharge

11

Valence
Electrons orbit the nucleus at certain distances from the
Electrons orbit the
nucleus at certain distances from the
Electrons
nucleus. Each orbit corresponds to a certain energy
nucleus. Each orbit corresponds to a certain energy
level.
level.
The electrons in the outer most orbit are called
The electrons in the outer most orbit are called
Valence
Valenceelectrons.
electrons.They
Theyhave
havehigher
higherenergy
energyand
and are
are
less
tightly
bound
to
the
atom
than
those
closer
to
the
less tightly bound to the atom than those closer to the
nucleus.
nucleus.
If valence electron acquires a sufficient amount of
If valence electron acquires a sufficient amount of
energy,
energy,ititcan
canescape
escapefrom
fromthe
theouter
outerorbit.
orbit.
The escaped valence electron is called a free
The escaped valence electron is called a free
electron.
electron.ItItcan
canmigrate
migrateeasily
easilyfrom
fromone
oneatom
atomto
to
another
anotheratom.
atom.
The outer orbit or
layer as it is usually
known as "valance
ring ".
As a
result, electron(s)
within the outer
orbit, it can be call
"valance
electron".
12

CONDUCTOR SEMI CONDUCTOR &


ISOLATOR
When electrons can move easily from
atom to
atom in a material, it is conductive. Metals
are good Conductors. They have a
large amount of
Free electrons . The good conductors
have
1-3 number
valance
electron.
A material
with a of
very
few free
electrons is
an Insulator. Insulators (dielectrics)
cannot conduct electricity but can hold or
store it. Insulators have 5-8 number of
valance electron.
Semiconductors conduct less than the
conductors but more than the insulators.
Because of their unique properties some of
them are the basis for modern electronic
devices such as diodes, transistors and
integrated circuits. Semiconductors have 4
13
number of valance electron

In a natural, or neutral state, each atom in a body of matter


will have
the proper number of electrons in orbit around it.
Consequently, the whole body of matter composed of the
neutral
atoms will also be electrically neutral. In this state, it is
said to have
a Zero Charge."
Electrons will leave or enter the neutrally charged body
should it come
in contact with other neutral bodies.
If, however, any number of electrons are removed from the
atoms of
a body of matter, there will remain more protons than
electrons and
the whole body of matter will become Electrically
Positive
14
Should the positively charged body come in contact with

Static Charge
A static charge is more easily created between non
conducting materials.
When a hard rubber rod is rubbed with fur, the rod will
accumulate
electrons given up by the fur.
When the charge becomes great enough, current will flow
regardless of
the poor conductivity of the materials.
These currents will cause Visible sparks and produce a
Crackling Sound

15

One of the Fundamental laws of electricity is that


Like Charges
Repel Each Other and Unlike Charges Attract
Each Other
A positive charge and negative charge, being unlike,
tend to move toward each other.

16

Coulomb's Law of Charges


The relationship between attracting or repelling
charged bodies was first discovered and written about
by a French scientist named Charles A. Coulomb.

Coulomb's Law states that


CHARGED BODIES ATTRACT OR REPELEACH OTHER
WITH A FORCE THAT IS DIRECTLY PROPORTIONAL TO
THE PRODUCT OF THEIR INDIVIDUAL CHARGES, AND
IS INVERSELY PROPORTIONAL TO THE SQUARE OF THE
DISTANCE BETWEEN THEM.
The amount of attracting or repelling force which acts
between two electrically charged bodies in free space
depends on two things:

1)Their charges and


2)The distance between them.
17

Electric Fields

The Space between and around charged bodies in which


their influence
is felt is called an ELECTRIC FIELD OF FORCE.
It can exist in air , glass , paper , or a vacuum.
ELECTROSTATIC FIELDS and DIELECTRIC FIELDS are
other names
used to refer to this region of force.
To avoid confusion, the lines of force exerted by a Positive
charge
are always shown leaving the charge, and for a Negative
charge they
are shown entering.
18

What Are the Hazards of Static


Electricity

The main hazard of static electricity is the creation


of sparks in an explosive or flammable Atmosphere
.These sparks can set off an explosion or fire. The
danger is greatest when flammable liquids are being
poured or transferred . For static electricity to be a
hazard, four conditions must be met :
1. There must be a means for a static charge to
develop.
2. Enough energy must build up to cause
ignition.
3. There must be a discharge of this energy (a
spark).
4.

The spark must occur in an Ignitable


vapors or

19

How is Static Generated ?


The main causes of static electricity
are:
1.Contact and Separation between two
materials (including friction, travelling over
rollers etc)
2.Rapid heat change (e.g. material going
through an oven)
3.High energy radiation, UV, x-ray, intense
electric fields (not very common in industry)
4.Cutting action (e.g a slitter or sheet cutter)
5.Induction (standing in the electric field
generated by
20
a static charge)

21

ENERGY
THE PRINCIPLE OF THE CONSERVATION OF ENERGY
Energy cannot be created or destroyed but can only be
transferred
from one body to another or changed from one form to
another.
In every chemical or physical reaction, there is no loss of
energy
any loss is always accompanied by an equal gain.

RENEWABLE & NON-RENEWABLE SOURCES OF


ENERGY
Every day we use energy for many purposes, e.g. transport
and heating.
The sources of this energy are many, e.g. oil, coal and
hydroelectricity.
Sources of energy are classified as renewable or non22
renewable

ELECTROMAGNETIC ENERGY
This is the energy transmitted from one point to another
in the form of
electromagnetic waves that travel at the speed of light.
Example of a
CHEMICAL
ENERGY
electromagnetic
energy producer is a Microwave.
The energy given out (or sometimes taken in) in the form
of heat,
light or kinetic energy during a chemical reaction is
called
chemical energy. For example, when something is
burned, some of the
ELECTRICAL
ENERGY
chemical energy
stored in it is changed into heat and
light When
energy.
an electric Current flows in a Metal wire
subatomic particles
called electrons move in the wire. As they move, they
lose energy
called electric potential energy. This potential energy 23
appears as

Electrical
Charges

Electrical charges are created by the displacement


of electrons
Consequently, a charge must always have either a
negative or
positive polarity .
A body with an Excess of Electrons is considered to
be
Negative, whereas a body with a Deficiency of
Electrons is
Positive.
A difference of potential can exist between two
points, or
bodies, only if they have different charges

24

Measuring the charges


Measuring charges is the Coulomb, named after
the scientist
Charles Coulomb.
One coulomb is equal to the charge of
6,280,000,000,000,000,000 (six quintillion two
hundred
and eighty quadrillion) or (6.28 x 10 ) electrons.
When a charge of one coulomb exists between two
bodies, one
unit of electrical potential energy exists, which
is called the
difference of potential between the two bodies.
This is referred to as ELECTROMOTIVE FORCE, or
25
VOLTAGE,

One body is deficient of 6 coulombs (representing 6 volts), and


Difference
the other is deficient
by 12 coulombs
(representing 12 volts),
of potential
there is a difference of potential of 6 volts.
The body with the greater deficiency is positive with respect
to the other.
When a difference in potential exists between two charged
bodies that
are connected by a conductor, electrons will flow along the
conductor.
This flow is from the Negatively charged body to the
Positively charged
body, until the two charges are equalized and the potential
difference
no longer exists

An Analogy of Potential
Difference

If the Voltage is
Increased, the Flow is
Increased.
If the Voltage is
Decreased, the Flow
is Decreased
26

Movement of electrons
Electrons in motion make up an electric current. This electric current is
usually referred to as "current" or "current flow
When the current flow is in one direction only, it is called direct current.
Current that reverses itself periodically, called alternating current.

To produce current, the electrons must be moved by a


potential difference.
The symbol for current is (I).
The basic measurement for current is the ampere (A).
One ampere of current is defined as the movement of one
coulomb of
I =
charge past any given point
of Current
a conductor during one second
( Ampere )
of time.
Q = Coulomb
t = Time
Current = Coulomb/Time
27

Measurement of Current
The magnitude of current is measured in AMPERES.
A current of one ampere is said to flow when one coulomb of
charge passes
a point in one second.
One coulomb is equal to the charge of 6.28 x 10 Electrons.
The device used to measure current is called an AMMETER

Conventional Current Flow versus


Electron flow

Conventional current was


defined early
in the history of electrical science
as flow
electric charge moves from the
positive
side of the power source to the
negative.
28
Electron flow can be said to flow

29


RESISTANCE
The property of a conductor of electricity that limits or restricts the
flow of electric current is called its resistance.
The conductors are materials examples of good conductors such as
copper, silver, and gold, chiefly metals, which possess a large
number of free electrons
Insulators are materials having few free electrons. Examples of
insulators are rubber, plastics, glass, and dry wood.
The unit used to measure resistance is called the ohm. The symbol
for the ohm is the Greek letter omega (). In mathematical
formulas, the capital letter "R" refers to resistance.
The resistance of a conductor and the voltage applied to it determine
the number of amperes of current flowing through the conductor.
Thus, 1ohm of resistance will limit the current flow to 1 ampere in a
conductor to which a voltage of 1 volt is applied

30

ELECTRICAL RESISTORS
Resistance is a property of every electrical component.
At times, its effects will be undesirable. However, resistance
is used in many varied ways. RESISTORS are
components manufactured to possess specific values of
resistance. They are manufactured in many types and
sizes

31

1. Fixed
2. Stepped or tapped
3. Variable

Type of
Resistor

1. Fixed Resistor
Symb
ol

Carbon and Metal Oxide Film


WireWound

32

Wire-wound resistors
are made with coils of
resistance wire.
Often enclosed in
ceramic to help dissipate
heat and protect the
resistor wire, they are
accurate and heat
stable.
The resistance value is
often marked.
Wire-wound are used in
higher watt circuits
often 2W or higher.

33

A stepped or tapped
resistor has two or more
fixed taps that provide
different resistance values.
These taps allow current
to flow through all or part
of the resistor, which
changes the amount of
current flowing through
the circuit.
Stepped resistors can also
be encased in ceramic and
are nothing more than a
series of fixed resistors
placed end to end.

34

3 . VARIABLE
RESISTORS
Variable resistors provide an infinite amount of
resistance values. Variable resistors are used by electrical
circuits .

1.
Potentiometer
s
2. Rheostats

Generic Variable Resistor Electrical


Symbol
35

POTENTIOMETER
Potentiometers have three connections or legs:
1.Reference ( R )
2.Signal ( S )
3. Ground. ( G )
The reference is at one end of a resistor and the Ground is at the other
end.
Current flows from the Reference through the resistor to Ground
creating a voltage drop across the resistor.
The Signal is a sliding contact (movable wiper arm) that runs across the
resistor.

The main purpose is


not to
vary resistance but to
vary
36
the voltage in a

Carbon Variable Resistor


Potentiometer

37

Rheostats at one time were used in the headlamp switch to


dim or brighten dash panel lighting
Rheostats have two connections, one to the fixed end of a
resistor and the other to a sliding contact on the resistor.
Turning the control moves the sliding contact away from or
toward the fixed end, increasing or decreasing the resistance.

Rheostats control
resistance,
thus controlling current
flow
38

Wire wound Resistor


(Rheostats )

39

Reading Resistor Value From Color


Coded
Bands
The color of the first band indicates the value of the first
significant digit.
The color of the second band indicates the value of the second
significant digit. The third color band represents a decimal
multiplier by which the first two digits must be multiplied to
obtain the resistance value of the resistor

40

41

The electrical resistivity can also be


Where
given by:
is the static resistivity (measured in ohm-metres , m)
R is the electrical resistance of a uniform specimen of the material
(measured in
ohms )
is the length of the piece of material (measured in metres , m)
A is the cross-sectional area of the specimen (measured in square
metres , m).
42

Factors That Affect Resistance


The magnitude of resistance is determined in part by the
"number of free electrons" available within the material. Since a
decrease in the number of free electrons will decrease the
current flow, it can be said that the opposition to current flow
(resistance) is greater in a material with fewer free electrons.
Thus, the resistance of a material is determined by the number
of free electrons available in a material. Good conductors will have
a low resistance

Affect the resistance of a conductor :


1.Type of material
2.Physical dimensions
3.Temperature
43

Type of Material ( Resistivity )


Depending upon their atomic structure,
different materials will have different
quantities of free electrons. Therefore, the
various conductors used in electrical
applications have different values of
resistance. Most metals are crystalline in
structure and consist of atoms that are
tightly bound . The atoms of such elements
are so close together that the electrons in
the outer shell of the atom are associated
with one atom as much as with its neighbor.
Materials of this type are poor conductors and
44
therefore have a high resistance.

Silver, gold, and aluminum are good


conductors. Therefore, materials
composed of their atoms would have a low
resistance.

The element copper is the conductor most


widely used throughout electrical
applications. Silver has a lower resistance
than copper but its cost limits usage to
circuits where a high conductivity is
demanded.
Aluminum, which is considerably lighter
than copper, is used as a conductor when
weight is a major factor.

45

EFFECT OF CROSS-SECTIONAL AREA


Cross-sectional
area
magnitude of resistance.

greatly

affects

the

If the cross-sectional area of a conductor is


increased, a greater quantity of electrons are
available for movement through the conductor.
Therefore, a larger current will flow for a given
amount of applied voltage.
If the cross-sectional area of a conductor is
decreased, the number of available electrons
decreases and, for a given applied voltage, the
current through the conductor decreases.
46

EFFECT OF CONDUCTOR LENGTH


The length of a conductor is also a factor which
determines the resistance of a conductor. If the
length of a conductor is increased, the amount of
energy given up increases. As free electrons move
from atom to atom some energy is given off as heat.
The longer a conductor is, the more energy is lost
to heat. The additional energy loss subtracts from the
energy being transferred through the conductor,
resulting in a decrease in current flow for a given
applied voltage. A decrease in current flow
indicates an increase in resistance, since voltage was
held constant. Therefore, if the length of a conductor
is increased, the resistance increases. THE
RESISTANCE
OF
A
CONDUCTOR
IS
DIRECTLY
47
PROPORTIONAL TO ITS LENGTH.

EFFECT OF
Temperature
changes affect the resistance of materials in
TEMPERATURE
different ways. In some materials an increase in
temperature causes an increase in resistance, whereas in
others, an increase in temperature causes a decrease in
resistance. The amount of change of resistance per unit
change in temperature is known as the TEMPERATURE
COEFFICIENT. If for an increase in temperature the
resistance of a material increases, it is said to have a
POSITIVE TEMPERATURE COEFFICIENT. A material whose
resistance decreases with an increase in temperature has
a NEGATIVE TEMPERATURE COEFFICIENT. Most conductors used
in electronic applications have a positive temperature
coefficient. However, carbon, a frequently used material, is
a substance having a negative temperature coefficient.
Several materials, such as the alloys constantan and
manganin , are considered to have a ZERO TEMPERATURE
COEFFICIENT because their resistance remains relatively
constant for changes in temperature.
48

Temperature changes affect


The Resistance R t at a temperature of t (C ) can be
calculated
From the approximation :
Rt = Ro ( 1 + t )
Where :
Ro is the Resistance at 0 C
is the temperature coefficient per degree, taking 0C as the standard

For example :
The field winding of a Generator has Resistance of 40 At 0 C .
What is its resistance at 50C ? , Resistance temperature
coefficient of Copper is 0.0043 per C at 0
Rt = Ro ( 1 + t )
= 40 ( 1 + 0.0043 X 50 )
= 40 X 1.215 = 48.6
49

50

HOW VOLTAGE IS
Presently,PRODUCED
there are six known methods for producing a
voltage or electromotive force (emf). Some of these methods
are more widely used than others, and some are used
mostly for specific applications. Following is a list of the six
known methods of producing a voltage:
1. FRICTION - Voltage produced by rubbing certain
materials together.
2. PRESSURE (Piezoelectricity) - Voltage produced by
squeezing crystals of
certain substances.
3. HEAT (Thermoelectricity) - Voltage produced by
heating the joint (junction)
where two unlike metals are joined.
4. LIGHT (Photo electricity) - Voltage produced by light
striking photosensitive
(light sensitive) substances.
51
5. CHEMICAL ACTION - Voltage produced
Prepared By by
: J uchemical
sri
reaction in

Voltage Produced by Friction


The development of charges by rubbing a rod with fur

is a prime example of the way in which a voltage is


generated by friction. Because of the nature of the
materials with which this voltage is generated, it cannot
be conveniently used or maintained. For this reason,
very little practical use has been found for voltages
generated by this method
Machines were developed in which charges were
transferred from one terminal to another by means of
rotating glass discs or moving belts. The most notable
of these machines is the Van de Graaff Generator. It is
used today to produce potentials in the order of millions
of volts for nuclear research. As these machines have
little value outside the field of research, their theory of
operation will not be described here.
52

Van de Graaff
Generator

Dr. Robert J. Van de Graaff developed the electrostatic


generator in 1929 while he was a research fellow at
Princeton University . He crafted a larger generator that
produced significantly higher voltages

53

Voltage Produced by Pressure


The first demonstration of the direct piezoelectric effect was
in 1880 by the brothers Pierre Curie and Jacques Curie the
piezoelectric effect causes crystal materials (like quartz)
to generate an electric charge when the crystal material
is compressed, twisted, or pulled. Quartz and Rochelle
salt exhibited the most piezoelectricity.

54

Naturally-occurring
crystals

Cane sugar
Quartz
Rochelle salt
Topaz
Tourmaline-group minerals
55

Applications
Direct piezoelectricity
of some substances like quartz, as
Piezoelectricity

mentioned above, can generate potential differences of


thousands of volts.
1.The best-known application is the electric cigarette lighter ;
pressing the button causes a spring-loaded hammer to hit a
piezoelectric crystal, producing a sufficiently high voltage
electric current that flows across a small spark gap thus
heating and igniting the gas.
2.Inkjet printers On many inkjet printers, piezoelectric crystals
are used to drive the ejection of ink from the inkjet print head
towards the paper
3.Piezoelectric disk used as a guitar pickup
4.Many rocket-propelled grenades used piezoelectric fuze .
For example: RPG-7

56

Voltage Produced by Heat


Thermocouples

have
somewhat
greater
power
capacities than crystals, but their capacity is still very
small if compared to some other sources. The
thermoelectric voltage in a thermocouple depends
mainly on the difference in temperature between the
hot and cold junctions. Consequently, they are widely
used to measure temperature, and as heat-sensing
devices in automatic temperature control equipment.

Voltage produced by heat


57

Voltage Produced by Light


When light strikes the surface of a substance, it may
dislodge electrons from their orbits around the surface
atoms of the substance. This occurs because light has
energy, the same as any moving force. Some substances,
mostly metallic ones, are far more sensitive to light than
others. That is, more electrons will be dislodged and
emitted from the surface of a highly sensitive metal,
with a given amount of light, than will be emitted from a
less sensitive substance. Upon losing electrons, the
photosensitive (light-sensitive) metal becomes positively
charged, and an electric force is created. Voltage
produced in this manner is referred to as a
PHOTOELECTRIC VOLTAGE.The photosensitive materials
most commonly used to produce a photoelectric voltage are
various compounds of silver oxide or copper oxide
58

The cell has a curved light-sensitive surface focused on the


central anode. When light from the direction shown strikes the
sensitive surface, it emits electrons toward the anode. The more
intense the light, the greater the number of electrons emitted.
When a wire is connected between the filament and the back, or
dark side of the cell, the accumulated electrons will flow to the
dark side.

59

Voltage Produced by Chemical Action


Voltage may be produced chemically when certain substances
are exposed to chemical action. If two dissimilar substances
(usually metals or metallic materials) are immersed in a solution
that produces a greater chemical action on one substance than
on the other, a difference of potential will exist between the
two. If a conductor is then connected between them, electrons will
flow through the conductor to equalize the charge. This
arrangement is called a primary cell. The two metallic pieces are
called electrodes and the solution is called the electrolyte

60

Voltage Produced by Magnetism


When a Conductor MOVE ACROSS a magnetic field so as to
cut the lines of force, electrons Within the conductor are
propelled in one direction or another. Thus, an electric force,
or voltage, is created.

61

62

BATTERIES

Batteries are widely used as sources of


direct-current
( DC ) electrical energy in automobiles,
boats,
aircraft,
ships,
portable
electric/electronic equipment, and lighting
equipment. In some instances, they are
used as the only source of power; while in
others, they are used as a secondary or
standby power source.
A battery consists of a number of cells
assembled in a common container and
connected together to function as a source
of electrical power.
63

Electrochemical Batteries
Electrochemical batteries consist of two or more
electrochemical cells. The cells use chemical reaction
to create a flow of electrons (electric current). Primary
elements of a cell include the container, two electrodes
(anode and cathode), and electrolyte material. The
electrolyte is in contact with the electrodes. Current is
created by the oxidation-reduction process involving
chemical reactions between the cells electrolyte and
electrodes.

64

Battery Discharges
When a battery discharges through a connected load,
electrically charged ions in the electrolyte that are
near one of the cells electrodes supply electrons
(oxidation) while ions near the cells other electrode
accept electrons (reduction), to complete the process.
The process is reversed to charge the battery, which
involves ionizing of the electrolyte.

65

THE CELL

A cell is a device that transforms chemical energy


into electrical energy. The simplest cell, known as
either a galvanic or voltaic cell It consists of two
electrodes a piece of carbon (C) and a piece of zinc
(Zn) suspended in a jar that contains a solution of
water (H2O) and sulfuric acid (H2SO4) called the
electrolyte

Simple voltaic or galvanic cell


66

ELECTRODES
The electrodes are the conductors by which the
current leaves or returns to the electrolyte. In the
simple cell, they are carbon and zinc strips that are
placed in the electrolyte ,while in the dry cell , they
are the carbon rod in the center and zinc container
in which the cell is assembled

Dry cell, cross-sectional view

67

ELECTROLYTE
The electrolyte is the solution that acts upon the
electrodes. The electrolyte, which provides a path for
electron flow, may be a salt, an acid, or an
alkaline solution. In the simple galvanic cell, the
electrolyte is in a liquid form. In the dry cell, the
electrolyte is a paste

CONTAINER
The container which may be constructed of one of
many different materials provides a means of
holding (containing) the electrolyte. The container
is also used to mount the electrodes. In the
voltaic cell the container must be constructed of a
material that will not be acted upon by the
electrolyte
68

PRIMARY CELL

A primary cell is one in which the chemical action eats


away one of the electrodes, usually the negative
electrode. When this happens, the electrode must be
replaced or the cell must be discarded. In the galvanictype cell, the zinc electrode and the liquid electrolyte are
usually replaced when this happens. In the case of the
dry cell, it is usually cheaper to buy a new cell.

SECONDARY CELL

A secondary cell is one in which the electrodes and the


electrolyte are altered ( berubah ) by the chemical
action that takes place when the cell delivers current.
These cells may be restored to their original condition
by forcing an electric current through them in the
direction opposite to that of discharge. the secondary
cell can be recharged and the electrodes are made of
different material
69

70

BATTERY CYCLING
The battery stores electricity
in the form of chemical
energy.
Through a chemical reaction
process the battery creates
and releases electricity as
needed by the electrical
system or devices.
Since the battery loses its
chemical energy in this
process, the battery must be
recharged by the alternator.
Reversing electrical current
flow through the battery the
chemical process is reversed,
thus charging the battery.
The cycle of discharging and
charging is repeated
continuously and is called
Battery Cycling".
71

Series-Connected Cells
Assume that a load requires a power supply of 6 volts
and a current capacity of 1/8 ampere. Since a single
cell normally supplies a voltage of only 1.5 volts, more
than one cell is needed. To obtain the higher voltage,
the cells are connected in series

72

Parallel-Connected
Cells
assume an electrical load requires

In this case,
only
1.5 volts, but will require 1/2 ampere of current.
(Assume that a single cell will supply only 1/8 ampere.)
To meet this requirement, the cells are connected in
parallel. There are four cells, so the total line current
is 1/8 x 4, or 1/2 ampere. In this case four cells in
parallel have enough capacity to supply a load
requiring 1/2 ampere at 1.5 volts.

73

Series-Parallel-Connected
Cells

Battery network supplying power to a load requiring


both a voltage and a current greater than one cell can
provide. To provide the required 4.5 volts, groups of three
1.5-volt cells are connected in series. To provide the
required 1/2 ampere of current, four series groups are
connected in parallel, each supplying 1/8 ampere of
current

74

BATTERY CONSTRUCTION
The lead-acid battery is one of the most common batteries in use
today and will be used to explain battery construction. The
container houses the separate cells. Most containers are hard
rubber, plastic, or some other material that is resistant to the
electrolyte and mechanical shock and will withstand extreme
temperatures. The container (battery case) is vented through
vent plugs to allow the gases that form within the cells to
escape. The cells are connected in series in the battery and the
positive terminal of one end cell becomes the positive
terminal of the battery. The negative terminal of the opposite
end cell becomes the negative terminal of the battery

75

Lead-acid Battery Plate


Arrangement
The terminals
of a lead-acid battery are usually identified from one

another by their size and markings. The positive terminal, marked


(+) is sometimes colored red and is physically larger than the
negative terminal, marked (-). The individual cells of the lead-acid
battery are not replaceable, so in the event one cell fails the
battery must be replaced

76

LEAD-ACID BATTERIES
CONSTRUCTION

77

LEAD-ACID BATTERIES
CONSTRUCTION
1. A

lead-acid cell
contains positive
plates coated with
lead peroxide (Pb02);
2. Negative plates made
of lead (Pb); and
3. A liquid electrolyte,
consisting of sulphuric
acid (H2SO4) and
water (H20).
78

LEAD ACID BATTERY CELL


ELEMENT

The key to battery operation is the cell


element.
Positive plates and negative plates are each
connected
together by separate plate straps.
These groups of positive and negative plates
are then
placed alternately, separated by microporous
separators.
Assembled together, the plates and
separators form a
battery cell element. Grouping the plates in
this way
serves to enlarge the surface area between
the active
79
materials and the electrolyte, thus allowing

CELL
VOLTAGE

Each cell element of the


battery produces
approximately 2.1
volts, regardless of the
quantity or size of the
plates. Automobile
batteries have six cells
that are connected in
series, which produces
a total voltage of 12.6
volts.

80

Nickel-Cadmium Battery
The nickel-cadmium battery is similar in construction to the
lead-acid battery with the exception that it has individual cells
which can be replaced. The construction of secondary cell
batteries is so similar, that it is difficult to distinguish the type of
battery by simply looking at it. The type of battery must be known to
properly check or recharge the battery. Each battery should have a
nameplate that gives a description of its type and electrical
characteristics

81

NICKEL-CADMIUM CELL
CONSTRUCTION
The nickel-cadmium battery
consists of :
1.Positive and Negative Plates,
The positive plates are made
from a porous plaque on which
nickel-hydroxide has been
deposited.
The Negative Plates are made
from similar plaques on which
cadmium-hydroxide is
deposited.
2. Separators
3. Electrolyte,
4. Cell vent
5. Cell container.

82

BATTERY CHARGING METHODS

The constant voltage Method


The constant-current method

83

Specific
The specific gravity of pure water is
1.0. Any substance thatGravity
sinks has a
specific gravity greater than 1.0.
The active ingredient in
electrolyte (sulfuric acid, potassium
hydroxide, etc.) is heavier than
water. Therefore, the electrolyte has
a SG greater than 1.0. The
acceptable range of specific gravity
for a given battery is provided by the
battery's manufacturer. To measure
a battery's SG, use an instrument
called a HYDROMETER.
SG Fully Charged per cell
approximately 1.260
Cell completely discharged 1.150

To read the hydrometer, hold it in a vertical position and read


the scale at the point that surface of the electrolyte touches the
float
84

Safety Precautions With

Batteries

All types of batteries should be handled with care:


1. NEVER SHORT THE TERMINALS OF A BATTERY.
2. CARRYING STRAPS SHOULD BE USED WHEN
TRANSPORTING BATTERIES.
3. PROTECTIVE CLOTHING, SUCH AS RUBBER APRON,
RUBBER GLOVES, AND A FACE SHIELD SHOULD BE
WORN WHEN WORKING WITH BATTERIES.
4. NO SMOKING, ELECTRIC SPARKS, OR OPEN FLAMES
SHOULD BE PERMITTED NEAR CHARGING BATTERIES.
5. CARE SHOULD BE TAKEN TO PREVENT SPILLING OF THE
ELECTROLYTE

85

CAPACITY AND RATING OF


BATTERIES
The CAPACITY of a battery is measured in amperehours. The ampere-hour capacity is equal to the
product of the current in amperes and the time in
hours during which the battery will supply this
current. The ampere-hour capacity varies inversely
with the discharge current. For example, a 400
ampere-hour battery will deliver 400 amperes for 1
hour or 100 amperes for 4 hours. Storage batteries
are RATED according to their rate of discharge and
ampere-hour capacity. Most batteries are rated
according to a 20-hour rate of discharge. That is, if a
fully charged battery is completely discharged
during a 20-hour period, it is discharged at the 20hour rate. Thus, if a battery can deliver 20 amperes
continuously for 20 hours, the battery has a rating
86
of 20 amperes x 20 hours, or 400 ampere-hours.

BATTERY
BatteriesCHARGING
are usually charged in battery
shops. Each shop will have specific charging
procedures for the types of batteries to be
charged. The following discussion will
introduce you to the types of battery charges.
The following types of charges may be given
to a storage battery, depending upon the
condition of the battery:
1. Initial charge
2. Normal charge
3. Equalizing charge
4. Floating charge
5. Fast charge
87

Initial Charge
When a new battery is shipped dry, the plates are in
an uncharged condition. After the electrolyte has
been added, it is necessary to charge the battery.
This is accomplished by giving the battery a long lowrate initial charge. The charge is given in accordance
with the manufacturer's instructions, which are
shipped with each battery.

Normal Charge
A normal charge is a routine charge that is given in
accordance with the nameplate data during the
ordinary cycle of operation to restore the battery to its
charged condition
88

Equalizing Charge
An equalizing charge is a special extended normal charge that
is given periodically to batteries as part of a maintenance
routine. It ensures that all the sulfate is driven from the plates
and that all the cells are restored to a maximum specific
gravity. The equalizing charge is continued until the specific
gravity of all cells, corrected for temperature, shows no change
for a 4-hour period

Floating Charge
In a floating charge, the charging rate is determined by the
battery voltage rather than by a definite current value. The
floating charge is used to keep a battery at full charge while the
battery is idle or in light duty. It is sometimes referred to as a
trickle charge and is accomplished with low current
89

Fast Charge
A fast charge is used when a battery must be
recharged in the shortest possible time. The charge
starts at a much higher rate than is normally used for
charging. It should be used only in an emergency, as
this type charge may be harmful to the battery.

Charging Rate

If the available charging equipment does not have


the desired charging rates, the nearest available rates
should be used. However, the rate should never be so
high that violent gassing (explained later in this text)
occurs

Charging Time
The charge must be continued until the battery is fully
charged. Frequent readings of specific gravity should
be taken during the charge and compared with the
reading taken before the battery was placed on charge.
90

Gassing
When a battery is being charged, a portion of the
energy breaks down the water in the electrolyte.
Hydrogen is released at the negative plates and
oxygen at the positive plates. These gases bubble up
through the electrolyte and collect in the air space at
the top of the cell. If violent gassing occurs when the
battery is first placed on charge, the charging rate is
too high. If the rate is not too high, steady gassing
develops as the charging proceeds, indicating that the
battery is nearing a fully charged condition.
WARNING
A mixture of hydrogen and air can be dangerously
explosive. No smoking, electric sparks, or open flames
should be permitted near charging batteries.
91

Thermocouples
The discovery of thermoelectricity is accredited to Thomas
Johann Seebeck (1770-1831) Raised in Estonia . In 1802 he
became a medical doctor. In 1821 he discovered that a
compass needle deflected when placed near a closed loop
formed from two dissimilar metal conductors if the junctions
were maintained at different temperatures. He also observed
that the deflection of the compass needle was proportional to
the temperature difference. Subsequently he described that it
depended on the type of conducting materials used, and not
on the temperature distributed along the conductors

Thomas Johann Seebeck


92

Thermocouple Theory
A thermocouple consists of two wires of dissimilar metals
joined together at one end, called the measurement ("hot")
junction. The other end, where the wires are not joined, is
connected called the reference ("cold") junction.

Use the terms "measurement junction" and "reference


junction" rather than the more traditional "hot junction"
and "cold junction." The traditional naming system can be
confusing
because
in
many
applications
the
measurement junction can be colder than the reference
junction
93

Thermocouple works with the principle of seebeck effect


Principle &
which states ;

Working
When two dissimilar
metal wires joined, are coupled
together and
exposed to difference in temperature at both the ends,
then emf is
generated and current flows from hot to cold side
(measurement to
reference junction) Temperature converts into electric
current
The magnitude of the current depends on the
temperature
difference between the junctions

94

Choosing a Thermocouple
When choosing a thermocouple consideration should be
given to both the thermocouple type, insulation and probe
construction. All of these will have an effect on the
measurable temperature range, accuracy and
reliability of the readings. Listed below is our
Type K (Chromel
thermocouple
types/ Alumel)
Type K is the general purpose thermocouple. It is low
cost and, its popularity, it is available in a wide variety of
probes. Thermocouples are available in the -200 C to
+1200 C range. Sensitivity is approx 41 V/C. Use
type K you have a good reason
Type E (Chromel / Constantan)
Type E has a high output (68 V/C) which makes it well
suited to low temperature (cryogenic) use. Another
property is that it is non-magnetic
95

Type J (Iron / Constantan)


Limited range (-40 to +750 C) makes type J less
popular than type K. The main application is with old
equipment that can not accept modern
thermocouples. J types should not be used above 760
C as an abrupt magnetic transformation will cause
permanent de calibration
Type N (Nicrosil / Nisil)
High stability and resistance to high temperature
oxidation makes type N suitable for high temperature
measurements without the cost of platinum (B,R,S)
types. Designed to be an improved type K, it is
becoming more popular
Type B (Platinum / Rhodium)
Suited for high temperature measurements up to 1800
C. Unusually type B thermocouples (due to the shape of
their temp./ voltage curve) give the same output at 0 C
and 42 C. This makes them useless below 50 C
96

Type R (Platinum / Rhodium)


Suited for high temperature measurements up to
1600 C. Low sensitivity (10 V/C) and high
cost makes them unsuitable for general purpose
use
Type S (Platinum / Rhodium)
Suited for high temperature measurements up to
1600 C. Low sensitivity (10 V/C) and high cost
makes them unsuitable for general purpose use. Due
to its high stability type S is used as the
standard of calibration for the melting point of
gold (1064.43C).
When selecting thermocouple types, ensure that
your measuring equipment does not limit the
range of temperatures that can be measured
97

98

99

Temperature
Applications
Exhaust Gas
Power Turbine
Compressor Discharge
Brake Systems
Bleed Air
Bearings
Lube/Hydraulic Oil
Fuel/Inlet Air

100

Thermocouple
Probe

101

Engine Oil Probe

102

103

THE BASIC ELECTRIC CIRCUIT


It contains a source of electrical energy (the dry cells in
the flashlight), a load (the bulb) which changes the
electrical energy into a more useful form of energy
(light), and a switch to control the energy delivered to
the load.
The LOAD is any device through which an electrical current
flows and which changes this electrical energy into a more
useful form. Some common examples of loads are a light bulb,
which changes electrical energy to light energy; an electric
motor, which changes electrical energy into mechanical
energy; and the speaker in a radio, which changes electrical
energy into sound.
The SOURCE is the device which furnishes the electrical energy
used by the load. It may consist of a simple dry cell (as in a
flashlight), a storage battery (as in an automobile), or a power
supply (such as a battery charger).
104
The SWITCH, which permits control of the electrical device,

SCHEMATIC DIAGRAM
The schematic diagram is a "picture" of the circuit that uses
symbols to represent the various circuit components; physically
large or complex circuits can be shown on a relatively small
diagram.

Symbols commonly used in electricity

105

Circuit, and the bulb (DS1) does not light. In figure B, switch S1
is closed. Current flows in the direction of the arrows from the
negative terminal of the battery (BAT), through the switch (S1),
through the lamp (DS1), and back to the positive terminal of the
battery. With the switch closed the path for current is complete.
Current will continue to flow until the switch (S1) is moved to the
open position or the battery is completely discharged

106

Three variables in a basic circuit

107

OHM'S LAW

In the early part of the 19th century, George Simon Ohm


proved by experiment that a precise relationship exists between
current, voltage, and resistance. This relationship is called
Ohm's law and is stated as follows:
The current in a circuit is DIRECTLY proportional to the
applied voltage and INVERSELY proportional to the circuit
resistance. As the resistance in a circuit increases, the
current decreases proportionately, Ohm's law may be
expressed as an equation:

108

APPLICATION OF OHM'S LAW


By using Ohm's law, you are able to find the resistance of a
circuit, knowing only the voltage and the current in the circuit.
In any equation, if all the variables (parameters) are known
except one, that unknown can be found. For example, using
Ohm's law, if current (I) and voltage (E) are known, resistance (R)
can be determined

Ohm's law in diagram form

109

If any two quantities are known, the third one can be


determined the schematic of the flashlight. If the
battery (BAT) supplies a voltage of 1.5 volts and the
lamp (DS1) has a resistance of 5 ohms, then the
current in the circuit can be determined. Using this
equation and substituting values

110

Determining resistance & Voltage in a basic cir

E = 10 volts
I = 1 ampere
R1 = ?

R1 = 45
I = 0.5 A
E=?
111

Current in a Series Circuit


Since there is only one path for current in a series circuit,
the same current must flow through each component of the
circuit.
The fact that the same current flows through each
component of a series circuit can be verified by inserting
meters into the circuit at various points, as shown in figure .
If this were done, each meter would be found to indicate the
Es = E source Et = E total
same value of current
Rt = R total
It = I total
Rt = R1 + R2 + R3 + R4
40 +30+20+10 = 100
Et
200
It = ----- It = ------ = 2 A
Rt
100
112

Voltage in a Series Circuit


The total voltage across a series circuit that consists
of more
than one resistor is also equal to the applied
voltage.
The sum of the individual resistor voltage drops.
In any series circuit, the sum of the resistor voltage
drops must
E1 = I1 x R1 = 2A x 40 = 80 V
equal the source voltage. E2 = I2 x R2 = 2A x 30 = 60 V
E3 = I3 x R3 = 2A x 20 = 40 V
E4 = I4 x R4 = 2A x 10 = 20 V

Et = E1 + E2 + E3 + .. En
Et = 80V + 60V + 40V + 20V = 200 V
113

Calculating individual voltage drops in a


series circuit

Ex = Voltage on
: R1 , R2 , R3
Rx = Resistance on : R1 , R2 , R3 ,
Rt = Total Resistance
Rx
Ex = x Es
Rt
40
E on R1 =
X 200 V = 80 V
100

114

Solving for applied voltage in a series c

Substituting:

115

VOLTAGE DIVIDERS
Voltage dividers are devices which make it possible to obtain
more than one voltage from a single power source. A voltage
divider usually consists of a resistor, or resistors connected in
series, with fixed or movable contacts and two fixed terminal
contacts. As current flows through the resistor, different
voltages can be obtained between the contacts.

116

To find total
resistance

To find the value of a resistor

To find total voltage

To find the voltage drop across a res

To find total current

To find current through a resistor

117

Resistors in series

If the resistors are placed in series then the overall resistance


is simply the
sum of the individual resistors.

R Total = R1 + R2 + R3
+ ...........

118

Resistors in parallel

If resistors are placed in parallel they share the current and


the situation
is a little more difficult to calculate, but still quite easy.

1/R Total

1/R1

1/R2

1/R3

+ ...............

Example
To give an example if there are three resistors in parallel with values
of 1 k ohm, 2 k ohm and 3 k ohm then the overall value of the
combination can be worked out:
1/R Total
1/R Total
1/R Total
1/R Total
R Total

=
=
=
=
=

1/1000 + 1/2000 + 1/3000


1/1000 + 1/2000 + 1/3000
6/6000 + 3/6000 + 2/6000
11/6000
6000/11 ohms = 545 ohms
119

Case of only two resistors in


parallel
Fortunately when there are only two
resistors in
parallel the formula simplifies to:
R Total = (R1 x R2) / (R1 + R2)
This may all seem rather mathematical,
but it is
all very useful when looking at electronic
circuits.

120

Current in Parallel
Resistance
parallel
circuit is one where the

A
ends of all the
devices are connected together. The resistors in the
following circuit are in parallel. Voltage is constant
in a parallel circuit. If the voltage constant across all
the resistors then we should be able to use Ohm's law
to calculate the current in each resistor . Current
drop in each resistor R1 , R2 and R3
I1 = V / R1
I2 = V / R2
I3 = V / R3
The total current in the circuit would
have to be the sum of all these
I = I1 + I 2 + I 3
This gives us
I = V (1/R1 + 1/R2 + 1/R3)
121

Real example :
V
R1
R2
R3

= 10 Volts
= 100
= 200
= 250

I1 = 10 V / 100 = 0.1 A or 100 mA


I2 = 10 V / 200 = 0.05 A or 50 mA
I3 = 10 V / 250 = 0.04 A or 40 mA
So the total current in the circuit would be...
0.19 A or 190 mA

If we calculate the total resistance with the formula that we derived we'd get...
1/R = 1/100 + 1/200 + 1/250
1/R = 0.019 R = 1/0.019 = 52.63
R = 52.63
Let's see if that get's us the current that we originally calculated.
I=V/R
I = 10V / 52.63
I = 0.19 A
So our equation works both ways.
122

Series-Parallel Circuits
A series-parallel circuit is a circuit that contains combinations of seriesconnected and parallel-connected components.
Two components are connected in series if they are connected to
each other at exactly one point and no other component is connected
to that point
Two components are connected in parallel if they are connected to
each other at two points
Circuit shown , R2 is in series with R3, and with R4. So R2, R3, and R4
form a series path. Also, this entire series path, taken as a whole, is in
parallel with R1.

123

Notation for Series-Parallel


Relationships
As a short-hand way of describing series-parallel relationships in a
circuit, we'll use two symbols:
We use the symbol + to indicate a series connection between
components
We use the symbol || to indicate a parallel connection
So for the example circuit shown above, we would write
R2 + R3 + R4 to indicate that R2, R3, and R4 form a series path.
Then, to show that R1 is in parallel with this series path, we would write

R1 || (R2 + R3 + R4)

IT = VS RT

124

Rparallel = 1/( 1/100 + 1/50 ) = 33.3

Using this result we can redraw the circuit


above as follows
now be calculated by summing the two series
resistances.

RT = 80 + 33.3 = 113.3

125

126

The basic unit of power


When voltage causes electrons to move, work is done. The
instantaneous Rate at which this work is done is called the
electric power rate, and is measured in WATTS. Power in
watts is equal to the voltage across a circuit multiplied by
current through the circuit. The symbol P indicates electrical
power. Thus, the basic power formula is P = E x I, where E is
voltage and I is current in the circuit. The amount of power
changes when either voltage or current, or both voltage and
current, are caused to change

127

Graph of power related to changing volt


The resistance of this circuit is 2 ohms; this value does not change.
Voltage (E) is increased (by increasing the voltage source), in steps of 1
volt, from 0 volts to 8 volts. By applying Ohm's law, the current (I) is
determined for each step of voltage. For instance, when E is 1 volt, the
current is:

128

P=ExI
P = 2 volts x 1 ampere
P = 2 watts
Or
2
= ---- P = 2 Watts
2
129

Graph of power related to changing curr


Thus, another formula for power, with
current and resistance as its factors,
is P = I2 x R. This can be proved by :

I = 3 Ampere , 4 Ampere
P = I2 x R

P = 32 x 2 = 18 A
P = 42 x 2 = 32 A
130

Summary of basic formulas


12 basic formulas The four quantities E, I, R, and P are at the
center of the figure. Adjacent to each quantity are three segments

131

For example
A circuit has a voltage source that delivers 6 volts and the circuit uses
3 watts of power. What is the resistance of the load?
The formula you would use is :

132

POWER RATING
The power rating, in watts, indicates the rate at
which the device converts electrical energy into
another form of energy, such as light, heat, or
motion. An example of such a rating is noted when
comparing a 150-watt lamp to a 100-watt lamp. The
higher wattage rating of the 150-watt lamp
indicates it is capable of converting more electrical
energy into light energy than the lamp of the lower
In
some electrical devices the wattage rating
rating
indicates the maximum power the device is
designed to use rather than the normal operating
power. For example, if a resistor normally used 2
watts of power, a resistor with a power rating of 3
watts
would
be
used.
133

Circuit for computing electrical quantities


P = 180 W , R = 5
E=?
I =?
E = P.R

180.5
= 30 Volt
I = E/R
I = 30/5 = 6 Ampere
Or
I = P/E
I = 180/30 = 6 Ampere

134

Power Consumption &

Efficiency
The term power consumption is common in the electrical field. It is

applied to the use of power in the same sense that gasoline


consumption is applied to the use of fuel in an automobile. Power
used by electrical devices is measured in energy. This practical unit
of electrical energy is equal to 1 watt of power used continuously
for 1 hour. The term kilowatt hour (kWh) is used more extensively on
a daily basis and is equal to 1,000 watt-hours
The EFFICIENCY of an electrical device is the ratio of power
converted to useful energy divided by the power consumed by the
device. This number will always be less than one (1.00) because of
the losses in any electrical device. If a device has an efficiency
rating of .95 it effectively transforms 95 watts into useful energy for
every 100 watts of input power. The other 5 watts are lost to heat, or
other losses which cannot be used
135

Horsepower, a unit of work, is often found as a rating on electrical


motors. One horsepower is equal to 746 watts. Example: A 3/4-hp
motor operates 8 hours a day. How much power is converted by the
motor per month? How many kWh does this represent?
Given:
t = 8 hrs x 30 days P = 3/4 hp
Solution:
Convert horsepower to watts
P = hp x 746 watts P = 3/4 x 746 watts P = 559 watts
Convert watts to watt-hours
P = work x time P = 559 watts x 8 x 30 P = 134,000 watt-hours
per month
(NOTE: These figures are rounded to the nearest 1000.)
136

Power in a Series Circuit


Since this power must come from the source, the total power
must be equal to the power consumed by the circuit
resistances. In a series circuit the total power is equal to the
SUM of the power dissipated by the individual resistors. Total
power (PT) is equal to:
PT = P1 + P 2 + P3 . . .Pn

By using the total resistance and the


applied voltage, the circuit current is
calculated

137

By means of the power formulas, the power can be calculated


for each resistor

total power delivered by the


source

The total power is equal to the sum of the power


used by the individual resistors.
138

Power in a Parallel Circuit


Power computations in a parallel circuit are essentially
the same as those used for the series circuit . The total
power is equal to the sum of the power dissipated by the
individual resistors.
I1 = Es/R1 = 50/10 = 5 Ampere
I2 = Es/R2 = 50/25 = 2 Ampere
I3 = Es/R3 = 50/50 = 1 Ampere

139

140

Measurement of
Capacitance
The basic
unit of measurement of capacitance is the

FARAD (F). A one-farad capacitor will store one


coulomb of charge (energy) when a potential of one
volt is applied across the capacitor plates. The farad
is an enormously large unit of capacitance. More
practical units are the micro farad (mF) or the pico
farad (pF).

141

Circuit symbols for capacitors


That each symbol is composed of two plates separated by a
space that represents the dielectric. The curved plate in
(B) of the figure indicates the plate should be connected to a
negative polarity

Non Polarized Capacitor Polarized Capacitor

Variable Capacitor
142

FACTORS AFFECTING THE VALUE OF


The value of capacitance of a capacitor depends
CAPACITANCE
on three factors:
The area of the plates.
The distance between the plates.
The dielectric constant ( Permittivity )of the
material between
the plates

143

PLATE AREA
A capacitor with the large plate area can store more
charges than
a capacitor with a small plate area. Simply stated, "the
larger the
plate area, the larger the capacitance".
DISTANCE BETWEEN THE PLATES
Electrostatic lines of force are strongest when the
charged
particles that create them are close together. When the
charged
particles are moved further apart, the lines of force
weaken, and the
ability to store charge decreases.
DIELECTRIC CONSTANT
The various insulating materials used as the dielectric
in
a capacitor differ in their ability to respond to (pass) 144

DIELECTRIC CONSTANT.
CONSTANT

A dielectric material with a high dielectric constant is a


better insulator than a dielectric material with a low
dielectric constant. Dielectric constants for some
common materials are given in the following list:
Material

Constant

Vacuum

1.0000

Air

1.0006

Paraffin paper

3.5

Glass

5 to 10

Mica

3 to 6

Rubber

2.5 to 35

Wood

2.5 to 8

Glycerine (15C)

56

Petroleum

Pure Water

81
145

CAPACITORS IN SERIES
The total capacitance of the circuit is developed between
the left plate of C1 and the right plate of C2. Because
these plates are farther apart, the total value of the
capacitance in the circuit is decreased. Solving for the
total capacitance (CT) of capacitors connected in series is
similar to solving for the total resistance(RT)of resistors
connected in parallel.

If the circuit contains only two


capacitors

Note : All values should be in farad


146

Example:
Determine the total capacitance of a series circuit containing
three capacitors whose values are 0.01 mF, 0.25 mF, and
50,000 pF, respectively

Note:
The total capacitance
of 0.008mF is slightly
smaller than the
smallest capacitor
(0.01mF).
147

CAPACITORS IN PARALLEL
When capacitors are connected in parallel, one plate of each
capacitor is connected directly to one terminal of the source,
while the other plate of each capacitor is connected to the
other terminal of the source. all the negative plates of the
capacitors connected together, and all the positive plates
connected together. Plate area equal to the sum of all the
individual plate areas. Connecting capacitors in parallel
effectively increases plate area and thereby increases total
capacitance.

148

Example:
Determine the total capacitance in a parallel
capacitive circuit containing three capacitors whose
values are 0.03 mF, 2.0 mF, and 0.25 mF, respectively

149

Capacitance Conversion

150

The Basic Physics Electromagnetism


The basic function of a relay
begins with the physics of
electromagnetism
The
knowledge of magnets and
electricity extends way back
in time but it wasn't until the
1800's that any connection
was made between the two.
In 1820 a Danish physicist named Hans Christian
rsted was the first to notice that an electrical current
produced a magnetic field (in this case it deflected a
compass placed near his battery). This discovery was
extended and formulated by Ampere, Faraday and
Maxwell. Joseph Henry invented the first relay in 1835.
In the simplest of terms this means that any wire that has
an electrical current passing through it will also have
a
151
magnetic field. The higher the current flow the


Electromagneti
Electromagnetism
smis produced when

an electrical
current flows through a simple conductor such as a
piece of wire or cable. A small magnetic field is
created around the conductor with the direction of this
magnetic field with regards to its "North" and "South"
poles being determined by the direction of the current
flowing through the conductor.

152

Left Hand Rule of Electromagnetism

Another more familiar concept which determines the


direction of current flow and the resulting direction of
the magnetic flux around the conductor is called the
"Left Hand Rule". By holding the current carrying
conductor in your left hand with the thumb extended it
will be pointing in the direction of the electron flow from
negative to positive. The position of the fingers laid
across the conductor will now point in the direction of the
magnetic lines of force as shown.

153

Electromagnetism
The current flowing through the two parallel conductors of
around
adirections
Loop as the current exits the
the loop are
in opposite
left hand side and returns on the right hand side. This
results in the magnetic field around each conductor inside
the loop being in the "SAME" direction to each other. The
resulting lines of force generated by the current flowing
through the loop oppose each other in the space between the
two conductors were the two like poles meet thereby
deforming the lines of force around each conductor as
shown.

154

Current in two Parallel Conductors


If current in two parallel conductors is flowing in
opposite directions, the magnetic fields would
also flow in opposite directions, and a natural
repulsion is created. The degree of repulsion
depends on the magnitude of the current

155

Magnetic Strength of Electromagnets


intensity of this field around the conductor is proportional

The
to the distance from it with the strongest point being next to
the conductor and progressively getting weaker further away
from the conductor. "Magnetic Field Strength", H sometimes
called "Magnetising Force" of a long straight current carrying
conductor is derived from the current flowing through it and the
distance from it.

H-is the strength of the magnetic field in ampereturns/metre, At/m


N-is the number of turns of the coil
I-is the current flowing through the coil in amps, A
L-is the length of the coil in metres, m

156

Electromagnetic Induction
Then the action of moving a coil or loop of wire
through a magnetic field induces a voltage in the coil
with the magnitude of this induced voltage being
proportional to the speed or velocity of the movement.
In other words the faster the movement of the magnetic
field the greater will be the induced emf or voltage in
the coil

157

Uses of Electromagnet Electric Bell


When the switch is on, the circuit is completed and
current
flows.
The
electromagnet
becomes
magnetized and hence attracts the soft-iron
armature and at the same time pull the hammer to
strike the gong. This enables the hammer to strike the
gong

158

Uses of Electromagnet Electromagnetic Relay


A relay is an electrical switch that opens and closes
under the control of another electrical circuit. The
switch is operated by an electromagnet to open or
close one or many sets of contacts. A relay has at least
two circuits. One circuit can be used to control
another circuit. The 1st circuit (input circuit) supplies
current to the electromagnet. The electromagnet is
magnetized and attracts one end of the iron armature

159

160

CHARACTERISTICS OF
INDUCTANCE

Inductance is the characteristic of an electrical


conductor that OPPOSES CHANGE in CURRENT. The
symbol for inductance is L and the basic unit of
inductance is the HENRY (H).

Inductance (L) is measured in


henrys (H). An inductor has an
inductance of one Henry (H) if
an emf of one volt is induced in
the inductor when the current
through
the
inductor
is
changing at the rate of 1
ampere per second. Common
units of inductance are Henry
(H),

161

ELECTROMOTIVE
FORCE (EMF)
Electromotive force is a difference of potential or voltage
which exists between two points in an electrical circuit. In
generators and inductors the emf is developed by the action
between the magnetic field and the electrons in a
conductor. Electromotive force is developed whenever there
is relative motion between a magnetic field and a
conductor

Generation of an emf in an electrical


conductor

162

Magnetic field & Induced Current

163

When a magnetic field moves through a


stationary metallic conductor, electrons are
dislodged from their orbits. The electrons move
in a direction determined by the movement of
the magnetic lines of flux. This is shown below

164

The electrons move from one area of the conductor


into another area. The area that the electrons moved
from has fewer negative charges (electrons) and
becomes positively charged. The area the electrons
move into becomes negatively charged. This is shown
below

The difference between the charges in the conductor


is equal to a difference of potential (or voltage). This
voltage caused by the moving magnetic field is
called electromotive force (emf).
165

In simple terms, the action of a moving magnetic field on


a conductor can be compared to the action of a broom.
Consider the moving magnetic field to be a moving
broom. As the magnetic broom moves along (through) the
conductor, it gathers up and pushes electrons before it,
as shown below:

The area from which electrons are moved becomes positively


charged, while the area into which electrons are moved
becomes negatively charged. The potential difference
between these two areas is the electromotive force or emf.
166

SelfInductance

The process by which a circuit induces an emf into itself by its


own moving magnetic field. All electrical circuits possess selfinductance. This opposition (inductance), however, only takes
place when there is a change in current. Inductance does NOT
oppose current, only a CHANGE in current. The property of
inductance can be increased by forming the conductor into a
loop. In a loop, the magnetic lines of force affect more of the
conductor at one time. This increases the self-induced emf.

167

Conductor Formed into a Loop or C


To increase the property of inductance, the conductor
can be formed into a loop or coil. Figure (A & B) shows a
conductor formed into a coil. Current through one loop
produces a magnetic field that encircles the loop in the
direction as shown in figure (3-A). As current increases,
the magnetic field expands and cuts all the loops as
shown in figure (3-B). The current in each loop affects all
other loops. The field cutting the other loop has the
effect of increasing the opposition to a current change

168

Inductors Classified

The inductor is made by forming a coil of wire


around a core. The core material is normally one of
two basic types: soft-iron or air core. The air-core
inductor may be nothing more than a coil of wire, but
it is usually a coil formed around a hollow form of
some nonmagnetic material such as cardboard to
hold the shape of the coil

Inductor types and schematic symbols


169

Affecting
Coil
Inductance
There
areFactors
several physical
factors
which
affect the
inductance of a coil. They include the number of turns in the
coil, the diameter of the coil, the coil length, the type of
material used in the core, and the number of layers of
winding in the coils. Figure shows two coils. Coil (A) has two
turns and coil (B) has four turns. In coil (A), the flux field set
up by one loop cuts one other loop. In coil (B), the flux field
set up by one loop cuts three other loops. Doubling the
number of turns in the coil will produce a field twice as
strong, if the same current is used. A field twice as strong,
cutting twice the number of turns, will induce four times the
voltage. Therefore, it can be said that the inductance varies
as the square of the number of turns

Inductance factor (turns).


170

view
Inductance
& diameter
Coil Diameter
The coil in
B has twice the
of coil view A.
Physically, it requires more wire to construct a coil of
large diameter than one of small diameter with an
equal number of turns. Therefore, more lines of force
exist to induce a counter emf in the coil with the larger
diameter. Actually, the inductance of a coil increases
directly as the cross-sectional area of the core
increases. Recall the formula for the area of a circle: A =
r2 Doubling the radius of a coil increases the
inductance by a factor of four.

Inductance factor
(diameter).

171

has
Inductance
Length
of spaced,
the Coil
Coil (A)
three turns,&rather
widely
making a relatively long coil. A coil of this type has few
flux linkages, due to the greater distance between
each turn. Therefore, coil (A) has a relatively low
inductance. Coil (B) has closely spaced turns,
making a relatively short coil. This close spacing
increases the flux
linkage, increasing the
inductance of the coil. Doubling the length of a coil
while keeping the same number of turns halves the
value of inductance.

Inductance factor (coil length). CLOSELY


WOUND

172

Inductance & Core Material Used


Coil (A) with an air core, and coil (B) with a soft-iron core.
The magnetic core of coil (B) is a better path for magnetic
lines of force than is the nonmagnetic core of coil (A). The
soft-iron magnetic core's high permeability has less
reluctance to the magnetic flux, resulting in more magnetic
lines of force. This increase in the magnetic lines of force
increases the number of lines of force cutting each loop of
the coil, thus increasing the inductance of the coil.

Inductance factor (core material). SOFTIRON CORE

173

Inductance & Wind the Coil in Layers

Shows three cores with different amounts of layering. The


coil in figure (A) is a poor inductor compared to the others
in the figure because its turns are widely spaced and there is
no layering. The important fact to remember, however, is that
the inductance of the coil increases with each layer added

Coils of various
inductances

X, are next to four


other turns
(shaded). This causes
the flux linkage to
be increased.
Y, lie directly next to
six other turns
174
(shaded).

Inductor
(i) Symbols
that flows through

The current
an inductor produces a
magnetic flux But unlike a Capacitor which oppose a change of
voltage across their plates, an inductor opposes the rate of
change of current flowing through it due to the build up of
self-induced energy within its magnetic field.

175

UNIT OF
INDUCTANCE
The basic unit of inductance (L) is the HENRY (H),
named after Joseph Henry, the co-discoverer with
Faraday of the principle of electro magnetic
induction. An inductor has an inductance of 1 henry if
an emf of 1 volt is induced in the inductor when the
current through the inductor is changing at the rate of
1 ampere per second. The relationship between the
induced voltage, the inductance, and the rate of
change of current with respect to time is stated
mathematically as :
Where : Eind is the induced emf in volts ; L is the inductance
in henrys; I is the change in current in amperes occurring
in t in seconds. The symbol & Delta; (Greek letter delta),
means "a change in .
176

Because the Henry is a relatively large unit of


inductance in its own right, for the smaller inductors
sub-units of the Henry are used to denote its value. For
example:
Prefix

Symbol

Multiplier

Power of Ten

milli

1/1,000

10-3

micro

1/1,000,000

10-6

nano

1/1,000,000,000

10-9

177

POWER LOSS IN AN
INDUCTOR
Copper Loss
Inductors are designed to carry relatively large amounts of
current power can be dissipated ( Loss ) in the inductor even
though the amount of resistance in the inductor is small. The
copper loss of an inductor can be calculated by multiplying the
square of the current in the inductor by the resistance of the
winding (I2R).

Hysteresis loss
Is due to power that is consumed in reversing the magnetic
field of the inductor core each time the direction of current in
the inductor changes.

Eddy-current loss
Is due to heating of the core by circulating currents that are
induced in the iron core by the magnetic field around the turns
of the coil. These currents are called eddy currents and circulate
within the iron core only
178

MUTUAL
Whenever two coils
are located so that the flux from one coil
INDUCTANCE

links with the turns of the other coil, a change of flux in one
coil causes an emf to be induced in the other coil. This allows
the energy from one coil to be transferred or coupled to the
other coil. The two coils are said to be coupled or linked by
the property of MUTUAL INDUCTANCE (M). The amount of
mutual inductance depends on the relative positions of the
two coils.

179

Two coils are placed close together as shown in figure .


Coil 1 is connected to a battery through switch S, and
coil 2 is connected to an ammeter (A). When switch S is
closed as in figure (A), the current that flows in coil 1
sets up a magnetic field that links with coil 2, causing an
induced voltage in coil 2 and a momentary deflection
of the ammeter. When the current in coil 1 . reaches a
steady value, the ammeter returns to zero. If switch S
is now opened as in figure (B), the ammeter (A) deflects
momentarily in the opposite direction, indicating a
momentary flow of current in the opposite direction in
coil 2. This current in coil 2 is produced by the
collapsing magnetic field of coil 1

180

FACTORS AFFECTING MUTUAL


INDUCTANCE
The mutual inductance of two adjacent
coils is dependent upon :
Physical dimensions of the two coils
The number of turns in each coil
The distance between the two coils
The relative positions of the axes of the
two coils
The permeability of the cores

181

182

DC Generator Theory
A device used to convert mechanical energy into electrical
energy is called as electric generator. Electrical generators
rotate a coil of wires ( wire is made into a loop) through a
magnetic field. AC generator uses slip rings to transfer the
current to the electrical circuit, while the DC generator uses a
split-ring called a Comutator to transfer the current to the
electrical circuit

AC generator

DC generator
183

MAGNETIC INDUCTION
When a conductor is moved in a magnetic field in such a
way that it cuts flux lines, and a voltage (emf) is induced in
the conductor. The Left Hand Rule for Generator states
that when the thumb, forefinger, and middle finger of the
left hand so that the thumb indicates the direction of
movement of the conductor in the magnetic field, and the
forefinger points in the direction of the flux lines (north to
south), the middle finger shows the direction of induced
emf in the conductor

184

AN ELEMENTARY GENERATOR
Elementary Generator consists of a single coil rotated in
a magnetic field. It produces an ac voltage.

185

BASIC DC GENERATOR
A BASIC DC GENERATOR results when you replace the slip
rings of an elementary generator with a two-piece
commutator , changing the output voltage to pulsating dc

Slip rings

Commutator

186

DC Circuits, that there are three conditions necessary to


induce a voltage into a conductor :
1. A magnetic field
2. A conductor
3. Relative motion between the two

187

MULTIPLE COIL ARMATURE


A MULTIPLE COIL ARMATURE (adding coils to the armature)
decreases the ripple voltage in the output of a dc
generator, and increases the output voltage

188

MULTIPOLE GENERATOR
A MULTIPOLE GENERATOR is the result of adding more
field poles to a dc generator. They have much the same
effect as adding coils to the armature. In practical generators,
the poles are electromagnets

189

ARMATURE TYPES
ARMATURE TYPES used in dc generators are the
Gramme-ring (seldom used) and the drum-type, used
in most applications

190

COMMUTATION
COMMUTATION is the process used to get direct current from
a generator. The coil connections to the load must be reversed
as the coil passes through the neutral plane. The brushes must
be positioned so that commutation is accomplished without
brush sparking

191

ARMATURE REACTION
ARMATURE REACTION takes place when armature current
causes the armature to become an electromagnet. The
armature field disturbs the field from the pole pieces. This
results in a shift of the neutral plane in the direction of
rotation

192

COMPENSATING WINDINGS AND


INTERPOLES
Compensating Winding & Inter Poles are used to
counteract the effects of armature reaction. They are
supplied by armature current and shift the neutral
plane back to its original position

193

MOTOR REACTION
MOTOR REACTION is caused by the magnetic field that
is set up in the armature. It tends to oppose the
rotation of the armature due to the attraction and
repulsion forces between the armature field and the
main field

194

ARMATURE LOSSES
In dc generator
armatures affect
ARMATURE
LOSSES
the outputs. These losses are as follows:
1.Copper losses are simply 12R (heat) losses caused by
current flowing
through the resistance of the armature windings.
2.
Eddy currents are induced in core material and
cause heat.
3.Hysteresis losses occur due to the rapidly changing
magnetic fields in
the armature, resulting in heat

Eddy Current in Armature Cores

195

FIELD EXCITATION
FIELD EXCITATION is the voltage applied
to the main field windings. The current in
the field coils determines the strength
and the direction of the magnetic field.
SEPARATELY EXCITED GENERATORS
receive
current for field coils from an outside
source
such, as a battery or an other dc
generator.
SELF-EXCITED GENERATORS use their
own
196
output voltage to energize field coils

SERIES-WOUND DC GENERATORS
SERIES-WOUND
DC
GENERATORS
have field
windings and armature windings connected in
series. Outputs vary directly with load currents.
Series-wound
generators
have
few
practical
applications

197

SHUNT-WOUND DC GENERATORS
SHUNT-WOUND DC GENERATORS have field
windings and armature windings connected in
parallel (shunt). The output varies inversely with
load current

198

COMPOUND-WOUND DC

COMPOUND-WOUND DC GENERATORS have both


GENERATORS
series
field windings and shunt field windings. These
generators combine the characteristics of series and
shunt generators. The output voltage remains
relatively constant for all values of load current within
the design of the generator. Compound generators are
used in many applications because of the relatively
constant voltage

199

Summary
A shunt-wound DC generator is constructed so that the
field winding is
in parallel with the armature winding
The voltage of a shunt-wound DC generator decreases
with an increase
in load current
A series-wound DC generator is constructed so that the
field winding is
in series with the armature winding
The voltage of a series-wound DC generator increases
sharply with an
increase in load
In a cumulatively-compounded DC generator, the
series and shunt
fields aid one another
In a differentially-compounded DC generator, the
series and shunt
fields oppose one another
The voltage of a flat-compounded DC generator
changes less than 5
200
percent from no-load to full-load

Output Voltage-vs-Load Current in DC Generato

Shunt-Wound

Series-Wound

Compounded
201

DC Generator
DC generator contains four ratings
Ratings

Voltage
Voltage rating of a machine is based on the insulation type
and design of the machine.
Current
The current rating is based on the size of the conductor and
the amount of heat that can be dissipated in the generator.
Power
The power rating is based on the mechanical limitations of the
device that is used to turn the generatorand on the
thermal limits of
conductors, bearings, and other components of the generator
Speed
Speed rating, at the upper limit, is determined by the
speed at
which mechanical damage is done to the machine. The
lower
speed rating is based on the limit for field current ( as
speed
202
increases, a higher field current is necessary to produce the

DIRECT CURRENT
INTRODUCTION

MOTORS

Many equipment depend on a dc motor for their power to


move. The speed
and direction of rotation of a dc motor are easily controlled.

useful
for
PRINCIPLES
OF OPERATION
operating
equipment,
as : winches,
missile
The operation
of a such
dc motor
is basedcranes,
on the and
following
launchers,
principle
:
which
must
moveconductor
in differentplaced
directions
and at varying
A current
carrying
in a magnetic
field,
speeds
perpendicular
to the lines of flux, tends to move in a direction
perpendicular to the
magnetic lines of flux

203

Right-hand rule for motors


Direction of current in the conductor, and the direction in
which the conductor tends to move. This relationship is
explained by using the RIGHT-HAND RULE FOR MOTORS

To find the direction of motion of a conductor, extend the thumb,


forefinger, Flux direction and middle finger Current direction
204

The loop of wire in the dc motor, however, has current flowing


through it from an external source. This current causes a
magnetic field to be produced

Speed of rotation of the coil can


be
increased by :
Increasing the number of turns
of the coil
Increasing the strength of the
current
Increasing the area of the coil
Increasing the strength of the
magnetic
field.
205

DC Motor Operation

206

DC Motor Construction
1. Ball bearings
2.
Alloy aluminum high
pressure die cast.
3.
Brushes accessible for
replacement,
4.
Copper winding
5.
Brass cartridge
6.
Molded Commutator
7.
Painted steel frame
8.
Armature/rotor
9.
Mounting includes foot

207

COUNTER
EMF
While a dc motor is running, it acts somewhat like a dc
generator. There is a magnetic field from the field
poles, and a loop of wire is turning and cutting this
magnetic field. For the moment there is current
flowing through the loop of wire from the battery. As
the loop sides cut the magnetic field, a voltage is
induced in them, the same as it was in the loop sides of
the dc generator. This induced voltage causes current
to flow in the loop. voltage is opposite that of the
battery, it is called Counter emf." This counter emf
cannot be equal to or greater than the applied battery
voltage
208

MOTOR
LOADS
Motors are used to turn mechanical devices, such as
water pumps, grinding wheels, fan blades, and
circular saws. For example, when a motor is turning a
water pump, the water pump is the load. The water
pump is the mechanical device that the motor must
move. This is the definition of a motor load As with
electrical loads, the mechanical load connected to a dc
motor affects many electrical quantities. Such things
as the power drawn from the line, amount of current,
speed, efficiency, etc., are all partially controlled by
the size of the load. The physical and electrical
characteristics of the motor must be matched to the
requirements of the load if the work is to be done
without the possibility of damage to either the load or
the motor
209

SERIES DC
MOTOR

Field is connected in series with the armature. The field


is wound with a few turns of large wire, because it
must carry full armature current. motor develops a
very large amount of turning force, called torque.
Because of this characteristic, the series dc motor can
be used to operate electric appliances, portable electric
tools, cranes, winches, APU Starter , etc. the speed
varies widely between no-load and full-load

210

SHUNT
MOTOR

The field windings are connected in parallel (shunt)


with the armature windings. the speed remains
relatively constant even under changing load
conditions. Armature current increases, causing the
motor to speed up. The conditions that established
the original speed are reestablished, and the
original speed is maintained.

211

COMPOUND
MOTOR

Field connected in parallel with the armature the other


is a series field that is connected in series with the
armature. The shunt field gives this type of motor the
constant speed advantage of a regular shunt motor. The
series field gives it the advantage of being able to
develop a large torque when the motor is started
under a heavy load. It should not be a surprise that the
compound motor has both shunt- and series-motor
characteristics

Prepared By : J u s r i

212

TYPES OF
ARMATURES

As with dc generators, dc motors can be constructed using


one of two types of armatures. Gramme-ring and Drumwound armatures
GRAMME-RING
ARMATURE

There are two parallel paths for current to follow - one up the
left side and one up the right side. These paths are completed
through the top brush back to the positive lead of the battery
213

DRUM-WOUND ARMATURE
The drum-wound armature is generally used in ac motors.
It is identical
to the drum winding on dc generators

The length of each conductor is positioned parallel to


the faces of the pole pieces. Therefore, each conductor
of the armature can cut the maximum flux of the motor
214
field.

DIRECTION OF
ROTATION

The direction of rotation of a dc motor depends on the


direction of the magnetic field and the direction of
current flow in the armature. If either the direction of
the field or the direction of current flow through the
armature is reversed, the rotation of the motor will
reverse. However, if both of these factors are reversed
at the same time, the motor will continue rotating in
the same direction. In actual practice, the field
excitation voltage is reversed in order to reverse
motor direction. Ordinarily, a motor is set up to do a
particular job that requires a fixed direction of
rotation. However, there are times when it is necessary
to change the direction of rotation, such as a drive
motor for a gun turret or missile launcher. Each of
these must be able to move in both directions. the
connections of either the armature or the field must be
215
reversed, but not both. In such applications, the proper

MOTOR
SPEED
A motor whose speed can be controlled is called a
variable-speed motor. The speed of a dc motor is
changed by changing the current in the field or by
changing the current in the armature

The rheostat speed control may be


connected either in parallel or in series
with the armature windings

216

ARMATURE
REACTION

The reasons for armature reaction and the methods of


compensating for its effects are basically the same for dc
motors as and dc generators. In DC Motor effect has shifted
the neutral plane backward, opposite direction of rotation.
This is different from a dc generator, where the neutral
plane shifted forward in the direction of rotation

Compensating windings and inter


poles are as important in motors as
they are in generators

217

Manual & Automatic Starter


Because the dc resistance of most motor
armatures is low (0.05 to 0.5 ohm), and
because the counter emf does not exist
until the armature begins to turn, it is
necessary to use an external starting
resistance in series with the armature of a
dc motor to keep the initial armature
current to a safe value. Controlling the
starting resistance in a dc motor is
accomplished either manually, by an
operator, or by any of several automatic
218
devices

219

What is alternating
current (AC)?

Direct Current (DC), which is electricity flowing in a


constant direction, and a voltage with constant
polarity. DC is the kind of electricity made by a battery
(with definite positive and negative terminals).
Sources of electricity (Rotary Electro-Mechanical
Generators) naturally produce voltages alternating in
polarity, reversing positive and negative over time , is
known as Alternating Current (AC)

DC and AC Comparison
220

Direct Versus Alternating Current


Whereas the familiar battery symbol is used as a
generic symbol for any DC voltage source, the
circle with the wavy line inside is the generic
symbol for any AC voltage source. Some cases AC
holds no practical advantage over DC. However,
with AC it is possible to build electric generators,
motors and power distribution systems that are
far more efficient than DC. AC voltage will be
produced across the wire coils as that shaft is
rotated, in accordance with Faraday's Law of
electromagnetic induction. This is the basic
operating principle of an AC generator, also known
as an alternator
221

AC Wave
Forms
alternator produces AC voltage,

When an
the voltage
switches polarity over time . When graphed over time, the
wave traced by this voltage of alternating polarity from an
alternator ,known as a sine wave

the change from one polarity to the other is a smooth one,


the voltage level changing most rapidly at the zero
(crossover) point and most slowly at its peak. If we were
to graph the trigonometric function of sine over a
horizontal range of 0 to 360 degrees we would find the
222
exact same pattern as in Table Next Page

Trigonometric sine function


Angle
( o)
0
15
30
45
60
75
90
105
120
135
150
165
180

sin(ang
wave
le)
0.0000 Zero
0.2588 +
0.5000 +
0.7071 +
0.8660 +
0.9659 +
1.0000 +Peak
0.9659 +
0.8660 +
0.7071 +
0.5000 +
0.2588 +
0.0000 zero

Angle
( o)
180
195
210
225
240
255
270
285
300
315
330
345
360

sin(ang
wave
le)
0.0000 Zero
-0.2588 -0.5000 -0.7071 -0.8660 -0.9659 -1.0000 -Peak
-0.9659 -0.8660 -0.7071 -0.5000 -0.2588 0.0000 zero
223


FREQUENC
The generator produces
one complete cycle of ac during each
second (1 Hz). IncreasingY
the number of revolutions to two
per second will produce two complete cycles of ac per second
(2 Hz). The number of complete cycles of alternating current
or voltage completed each second is referred to as the
FREQUENCY. Frequency is always measured and expressed in
hertz.

224


An individual cycle of any sine wave represents a definite
PERIOD
amount of TIME. Shows 2 cycles of a sine wave which has
a frequency of 2 hertz (Hz). Since 2 cycles occur each
second, 1 cycle must require one-half second of time. The
time required to complete one cycle of a waveform is
called the PERIOD of the wave. In figure , the period is onehalf second. The relationship between time (t) and
frequency (f) is indicated by the formulas

225

Each cycleofShaped
the sine wave
shown in figure
Variations
before consists of two identically shaped
variations in voltage. The variation which
occurs during the time the voltage is
positive
is
called
the
POSITIVE
ALTERNATION. The variation which occurs
during the time the voltage is negative is
called the NEGATIVE ALTERNATION. In a sine
wave, these two alternations are identical in
size and shape, but opposite in polarity. The
distance from zero to the maximum value of
each alternation is called the AMPLITUDE.
The amplitude of the positive alternation
and
the
amplitude
of
the
negative
226
alternation are the same.

WAVE
The distance traveled
by the sine wave during this
LENGTH
period is referred to as WAVELENGTH. Wavelength,
indicated by the symbol (Greek lambda), is the
distance along the waveform from one point to the same
point on the next cycle. The point on the waveform that
measurement of wavelength begins is not important as
long as the distance is measured to the same point on the
next cycle

227

Period of a Wave in Degrees


The period of a wave in degrees is always 360 . A more
popular measure for describing the alternating rate of an
AC voltage or current wave than period is the rate of
that back-and-forth oscillation. This is called frequency.

Alternator voltage as function of shaft position ( time )


An instrument called an oscilloscope, is used to display a
changing voltage over time on a graphical screen
228

Time period of Sine Wave is shown on


Oscilloscope
Period and Frequency is very useful to know when
displaying an AC voltage or current waveform on an
oscilloscope screen. By measuring the period of the wave on
the horizontal axis of the oscilloscope screen and
reciprocating that time value (in seconds), you can
determine the frequency in Hertz.

229

Other
Waveforms
electromechanical
alternators

While
naturally produce
sine waves . Other Waveforms of AC are commonly
produced within electronic circuitry. Here are but a few
sample waveforms and their common designations in
figure below

230

Measurements of AC
magnitude

AC voltage alternates in polarity and AC current alternates in


direction. We also know that AC can alternate in a variety of
different ways, and by tracing the alternation over time we
can plot it as a Waveform. We can measure the rate of
alternation by measuring the time it takes for a wave before
it repeats itself (the period), and express this as cycles per
unit time, or frequency. Magnitude (also called the
amplitude), of an AC quantity is to measure its peak height
on a waveform graph. This is known as the peak or crest
value of an AC waveform

Peak voltage of a waveform


231

~ ALTERNATING CURRENT
VALUES ~

In discussing Alternating Current and Voltage, we will often


find it necessary to express the Current and Voltage in terms
of Maximum or Peak Values, Peak-to-Peak Values,
Effective values, Average Values, or Instantaneous
Values. Each of these values has a different meaning and is
used
to describe
a different amount of current or voltage
PEAK
AND PEAK-TO-PEAK

VALUES
At this point the DC and AC Values are equal. This point
on the sine wave is referred to as the maximum or peak
value.

232

During each complete cycle of AC there are always two


maximum or peak values, one for the Positive half-cycle and
the other for the Negative half-cycle. The difference between
the peak positive value and the peak negative value is
called the Peak-to-Peak Value of the sine wave. Usually
Alternating Voltage and Current are expressed in

Effective Values

Another way is to measure


the total height between
opposite peaks. This is
known as the peak-to-peak
(P-P) value of an AC
waveform
233

INSTANTANEOUS
VALUE
The value may be zero if the particular instant is the time in
the cycle at which the polarity of the voltage is changing. It
may also be the same as the peak value, if the selected
instant is the time in the cycle at which the voltage or
current stops increasing and starts decreasing. There are
actually an infinite number of instantaneous values
between zero and the peak value

The positive points cancel out all the negative points over
a full cycle
The average value of a sine wave is zero
234

AVERAGE
VALUE

The Average Value of an Alternating Current or Voltage is


the average of all the Instantaneous Values during One
alternation. Since the voltage increases from zero to peak
value and decreases back to zero during one alternation,
the average value must be some value between those two
limits. You could determine the average value by adding
together a series of instantaneous values of the
alternation (between 0 and 180), and then dividing the
sum by the number of instantaneous values used. The
computation would show that one alternation of a sine wave
has an average value equal to 0.636 times the peak value.
The formula for average voltage is ;

E average = 0.636 X E max


Where E average is the average voltage of one alternation,
and E max is the maximum or peak voltage.
235

Similarly, the formula for average current is ;

I average = 0.636 X I max


where I average is the average current in one alternation, and Imax
is the maximum or peak current.
Do not confuse the above definition of an average value with that
of the average value of a complete cycle. Because the voltage is
positive during one alternation and negative during the other
alternation, the average value of the Voltage Values
occurring during the complete cycle is Zero.
However, as a practical
measure of
a waveform's average is
usually defined as the
mathematical mean. In other
words, we calculate the
practical average value of the
waveform by considering all 236
points on the wave as positive

EFFECTIVE VALUE OF A SINE


WAVE

E max , E avg, I max, and I avg


are values used in ac
measurements. Another value used is the Effective Value of
ac This is the Value of alternating Voltage or Current that
will have the same effect on a resistance as a comparable
value of DC Voltage or Current will have on the same
resistance.
When direct current flows in a resistance, the amount of
electrical power converted into heat equals I2R watts.
However, since an alternating current having a maximum
value of 1 ampere does not maintain a constant value, the
alternating current will not produce as much heat in the
resistance as will a direct current of 1 ampere

Heating effect of AC and DC.

237

Examine views A and B and notice that the heat (70.7 C) produced
by 1 ampere of alternating current (that is, an ac with a
maximum value of 1 ampere) is only 70.7 percent of the heat
(100 C) produced by 1 ampere of Direct Current. Mathematically

Therefore, for effective value of ac (I eff ) =


0.707 X I max.

The Effective Value of Alternating Current, and is known as


the Heating Effect" method. An alternating current is said to
have an effective value of one ampere when it produces heat
in a given resistance at the same rate as does one ampere of
Direct Current
238

The Effective Value is often called the Root-MeanSquare" (RMS) value

You can compute the


effective value of a sine
wave of current to a fair
degree of accuracy by taking
equally-spaced
instantaneous values of
current along the curve and
extracting the square root
of the average of the sum of
the squared values.
For this reason, the effective
value is often called the
"root-mean-square" (rms)
value
239

Stated another way, the Effective or RMS Value of a sine


wave of current

I eff = 0.707 X Imax.


When I eff is known, you can find Imax by using the formula

Imax = 1.414 X Ieff.


Where the constant 1.414 comes from.
Assume that the DC is maintained at 1 ampere and the
resistor temperature at 100C. Also assume that the AC in
is increased until the temperature of the resistor is 100 C.
At this point it is found that a maximum AC Value of 1.414
amperes is required in order to have the same heating
effect as Direct Current.
Remember the above relationship and that the effective
value (I eff) of any sine wave of current is always 0.707 times
the maximum value (Imax).
240

RMS Amplitude Measurement


RMS (Root Mean Square) amplitude measurement is the best
way to relate AC quantities to DC quantities, or other AC
quantities of differing waveform shapes, when dealing with
measurements of electric power. For other considerations, peak
or peak-to-peak measurements may be use . For instance, when
determining the proper size of wire to conduct electric power
from a source to a load, RMS current measurement is the best to
use

241

Comparing two different types of wave


Waveform amplitude when comparing two different types of
waves. For example, a square wave peaking at 10 volts is
obviously a greater amount of voltage for a greater amount of
time than a triangle wave peaking at 10 volts. The effects of
these two AC voltages powering a load would be quite
different : Figure below

A square wave produces a greater heating


effect than the same peak voltage triangle
wave

242

Band saw & jig saw analogy


of DC vs AC
Changing quantities of AC voltage or current measurement
present in this saw analogy: A band saw blade moves with
a constant speed, similar DC current moves with a constant
magnitude. A jig saw blade moves back and forth, its blade
speed constantly changing ( AC ). Picture a jigsaw and
band saw , equipped with identical blades (same tooth
pitch, angle, etc.), equally capable of cutting the same
thickness of the same type of wood at the same rate. We
might say that the two saws were equivalent or equal in
their cutting capacity

243

RMS Voltage Produces the same


Heating effect as a the same DC
Voltage

Two circuits above, we have the same amount of load


resistance (2 ) dissipating the same amount of power
in the form of heat (50 watts), one powered by AC and
the other by DC. Because the AC voltage source pictured
above is equivalent (in terms of power delivered to a
load) to a 10 volt DC battery, we would call this a 10244

Voltage Wave
Forms

We now know that there are


two types of Current and
Voltage, that is, Direct
current and Voltage and
Alternating current and
Voltage. View A. The DC
voltage is shown to have a
Constant amplitude. Some
voltages go through periodic
changes in amplitude like
those shown in view B. The
pattern which results when
these
changes
in
amplitude with respect to
time are plotted on graph
paper
is
known
as
a
WAVEFORM.
View
B
shows some of the common
electrical waveforms.

245

Simple AC circuit
calculations
AC circuit measurements and calculations can get very
complicated due to the complex nature of alternating current in
circuits with inductance and capacitance. However, with
simple circuits involving nothing more than an AC power
source and resistance, the same laws and rules of DC apply
simply
and directly
AC circuit
calculations for resistive circuits are the
same as for DC

246

AC
Phase

Out of phase
waveforms

Two or more AC Voltages or Currents that are out of step


with each other. By out of step, mean that the two
waveforms are not synchronized: that their peaks and
zero points do not match up at the same points in time.
The two waves shown the graph (A versus B) are of the same
amplitude and frequency, but they are out of step with
each other. In technical terms, this is called a phase shift.

247

Wave A leads wave B


by 45o

Calculating the trigonometric sine function for angles


ranging from 0 to 360 , a full circle. The starting point
of a sine wave was zero amplitude at zero degrees,
progressing to full positive amplitude at 90 degrees, zero at
180 degrees, full negative at 270 degrees, and back to
the starting point of zero at 360 degrees. We can use this
angle scale along the horizontal axis of our waveform plot to
express just how far out of step one wave is with another:
Figure below

Prepared By : J u s r i

248

Voltage waves 90 out of


phase
.
The terms
LEAD and LAG are used. The amount by which
one sine wave leads or lags another sine wave is measured
in degrees. You can also describe this relationship by saying
that wave E1 leads wave E 2 by 90, or that wave E2 lags
wave E 1 by 90. Whichever waveform is ahead in its
evolution is said to be leading and the one behind is said to
be lagging.

249

Voltage and Current waves in phase.


To be in phase, the two sine waves must go through their
maximum and minimum points at the same time and in the
same direction. The voltage increases in a positive direction,
the current increases along with it, and that when the voltage
reverses direction, the current also reverses direction are
precisely in step with one another, they are said to be in
phase

250

Voltage waves 180 out


of The
phase
two waves differ in phase by 180.The waves pass through
their maximum and minimum values at the same time, their
instantaneous voltages are always of opposite polarity. If two
such waves exist across the same component, and the waves are
of equal amplitude, they cancel each other.

251

252

Inductance & Alternating

Current
When the Current and Voltage are in step, going through a
cycle falling together and rising together, they are in phase.
When the angle of lead or lag they are out of phase . The
values leads or lags the other-is a measure in degrees of the
amount they are out of step. The current through the inductor
always lags the voltage across the inductor
A circuit having pure resistance would have the alternating
current through it the Voltage & Current across it rising and
failing together. The current and voltage do not have the
same amplitude, but they are in phase

253

Voltage & Current Waveforms in an


inductive
circuit
The wave forms
for a purely inductive AC circuit in

steps of
quarter-cycles. With an ac voltage, in the first quarter-cycle
(0 to 90) Since inductance opposes any change in current
flow, no current flows during the first quarter-cycle. In the
next quarter-cycle (90 to 180) the voltage decreases back
to zero; current begins to flow in the circuit and reaches a
maximum value at the same instant the voltage reaches zero
( Cycle ). The applied voltage now begins to build up to
maximum in the other direction, to be followed by the resulting
current.

254

When the voltage again reaches its maximum at the end of


the third quarter-cycle (270) all values are exactly
opposite to what they were during the first half-cycle. The
applied voltage leads the resulting current by one
quarter-cycle or 90 degrees. To complete the full 360
cycle of the voltage, the voltage again decreases to zero
and the current builds to a maximum value

Both Current and Voltage are always


changing in amplitude and direction.
255

Comparison of sine wave and circle


in an inductive circuit
The sine wave can be compared to a circle. Just as you mark
off a circle into 360 degrees, you can mark off the time of one
cycle of a sine wave into 360 electrical degrees. We can see
why the current is said to lag the voltage, in a purely inductive
circuit, by 90 degrees. In a circuit having both resistance and
inductance then, as you would expect, the current lags the
voltage by an amount somewhere between 0 and 90 degrees

256

INDUCTIVE
REACTANCE
current flowing through

When the
an inductor continuously
reverses itself, as in the case of an ac source, the inertia effect
of the cemf is greater than with dc. This opposing force which an
inductor presents to the Flow of alternating current cannot
be called resistance, since it is not the result of friction within
a conductor. The name given to it is Inductive Reactance
because it is the "reaction" of the inductor to the changing value
of alternating current. Inductive reactance is measured in ohms
and its symbol is XL Reactance increases with an increase of
frequency and with an increase of inductance. The formula for
The following example problem illustrates
inductive reactance is as follows
the computation of X .
L

257

Capacitors & Alternating


theCurrent
beginning of the first quarter-cycle (0 to

At
90) the
voltage passed through zero and is increasing in the
positive direction , charge on a capacitor with the voltage
.The greatest number of electrons are moving off one plate
and onto the other plate. Capacitor current maximum
value(figure A) . As the voltage proceeds toward maximum at
90 degrees, its rate of change becomes less and less, the
current decrease to zero. At 90 degrees the voltage
maximum, the capacitor is fully charged, and there is no
further movement of electrons from plate to plate. That is
why the current at 90 degrees is zero.

258

Second quarter-cycle (90 to


At the end 180).
of this first quarter-cycle the alternating voltage
stops increasing in the positive direction and starts to
decrease. It is still a positive voltage, but capacitor decrease
in voltage means that the plate which has accumulated an
excess of electrons must lose some electrons. The current
flow, reverse its direction ( figure B ) shows the current
curve to be below the zero line (negative current direction)
during the second quarter-cycle (90 to 180). At 180
degrees the Voltage has dropped to zero. This means that
the electrons are equally distributed between the two
plates; the Current is maximum ( Negative )

259

Third quarter-cycle (180 to 270).


After 180 degrees the Voltage has reversed polarity and
starts building up its maximum negative peak which is
reached at the end of the third quarter-cycle (180 to 270).
During this third quarter-cycle the rate of voltage change
gradually decreases as the charge builds to a maximum at
270 degrees. At this point the capacitor is fully charged
and it carries the full impressed voltage. The current flow
is zero at this point. The conditions are exactly the same as at
the end of the first quarter-cycle (90) but the polarity is
reversed

260

Quarter-cycle (270 to 360)


After 270 degrees the impressed voltage once again starts to
decrease, and the capacitor must lose electrons from the
negative plate. It must discharge, starting at a minimum rate
of flow and rising to a maximum. This discharging action
continues through the last quarter-cycle (270 to 360) until the
impressed-voltage has reached zero. At 360 degrees back at
the beginning of the entire cycle. This time and place
relationship between the current and voltage is called the
phase relationship. The voltage-current phase relationship in
a capacitive circuit is exactly opposite to that in an inductive
circuit. The current of a capacitor leads the voltage across
the capacitor by 90 degrees.

261

The Alternating Current flows "through" the capacitor is


not true
The amplitude and polarity of the voltage and the
The current
amplitude
and swing from the positive peak at zero degrees
to direction
the
of the current are continually changing.
negative peak at 180 degrees is NOT a measure of the
number of
electrons, or the charge on the plates. It is a picture of
the direction
and strength of the current in relation to the polarity and
strength of
Actually,
theappearing
electrons do
not the
pass
through
the dielectric,
the voltage
across
plates
.
but their
rushing back and forth from plate to plate causes a
current flow in
A capacitor will appear to conduct an alternating
the circuit
current and
a capacitor will not conduct a direct current

262

CAPACITIVE
REACTANCE
The capacitor as a device which the only opposition to the
alternating current has been the normal circuit resistance
present in any conductor. However, capacitors themselves
offer a very real opposition to current flow. This opposition
arises from the fact that, at a given voltage and frequency,
the number of electrons which go back and forth from plate
to plate is limited by the storage ability-that is, the
capacitance-of the capacitor. Increasing the frequency will
also decrease the opposition by a capacitor. This opposition
is called CAPACITIVE REACTANCE.
Capacitive Reactance decreases with increasing frequency
or

Capacitive reactance decreases with increasing


capacitance.
The symbol for Capacitive Reactance is XC XC varies inversely
with the product of the frequency and capacitance. The formula is:
263

Where:
XC is capacitive reactance in ohms f is frequency in
Hertz C is capacitance in farads p is 6.28 (2 X
3.1416) The following example problem illustrates the
computation of X C

264

Reactance, Impedance & Power


Relationship in AC Circuits
The combination of inductance, capacitance, and
resistance in AC Circuits. To explain the various properties
that exist within ac circuits, will be used the schematic
diagram of the series RLC circuit. The symbol shown in that is
marked E is the general symbol used to indicate an ac
voltage source

265

REACTANCE
The effect of Inductive
reactance is to cause the
current to lag the voltage, while that of
Capacitive reactance is to cause the current to
lead the voltage. Therefore, since inductive
reactance and capacitive reactance are exactly
opposite in their effects, what will be the result
when the two are combined? It is not hard to see
that the net effect is a tendency to cancel each
other, with the combined effect then equal to the
difference between their values. This resultant is
called REACTANCE represented by the symbol X
and expressed by the equation X = XL XC or X =
X C - X L.

266

For a practical example, suppose you have a circuit containing an


inductor of 100 mH in series with a capacitor of .001 mF,
and operating at a frequency of 4 MHz. What is the value of
net reactance, or X?

267

A circuit containing a 100 - mH inductor in series with a .0002mF capacitor, and operating at a frequency of 1 MHz. What is
the value of the resultant reactance in this case?

When capacitive and inductive reactance are combined in


series, the smaller is always subtracted from the larger and
the resultant reactance always takes the characteristics of
268
the larger

IMPEDANCE
Inductive reactance and Capacitive reactance act to oppose
the flow of current in an AC circuit. However, there is another
factor, the resistance, which also opposes the flow of the
current.
AC circuits containing reactance also contain resistance, the two
combine to oppose the flow of current
This combined opposition by the resistance and the reactance is
called the IMPEDANCE , and is represented by the symbol Z
The values of resistance and reactance are both given in ohms
AC circuit which contains only resistance, the current and the
voltage will be in step ( In phase ). Current and voltage do not have
the same amplitude, but they are in phase
AC circuit containing only reactance the current will either lead
( XC ) or lag ( XL ) the voltage by one-quarter of a cycle or
90 degrees.

269

Find the impedance of a series combination


Resistance and Reactance cannot simply be added together
to find the impedance, or Z, Because the current through a
resistor is in step with the voltage across it and the current in
a reactance differs by 90 degrees from the voltage across it,
the two are at right angles to each other. They can therefore be
combined by means of the same method used in the
construction of a right-angle triangle
Start by drawing a horizontal
line, R, representing 8 ohms as
the base of the triangle. Then,
since the effect of the reactance
is always at right angles, or 90
degrees, resistance, draw the
line XL, representing 5 ohms
inductive reactance, as the
altitude of the triangle.
270

The properties of a right


triangle

271

Vector diagram of resistance, capacitive


reactance, and impedance in a series
circuit
To draw the line representing
the Capacitive Reactance (XC)

in
a downward direction. To indicate that it acts in a direction
opposite to Inductive Reactance (XL) which is drawn upward.
In a series circuit containing capacitive reactance the
equation for finding the impedance becomes:

272

Vector diagram of Resistance, Reactance ( XL &


XC ) and Impedance ( Z ) in a series circuit
Assume that 10 ohms inductive reactance and 20 ohms capacitive
reactance are connected in series with 40 ohms resistance. Let the
horizontal line represent the resistance R. The line drawn upward
from the end of R, represents the inductive reactance, XL. Represent
the capacitive reactance by a line drawn downward at right angles
from the same end of R. The resultant of XL and XC is found by
subtracting XL from XC. This resultant represents the value of X.

273

The line, Z, will then represent the resultant of R and X. The


value of Z can be calculated as follows:

274

OHMS LAW FOR AC


In general, Ohm's law cannot be applied AC Circuits since it
does not consider the reactance which is always present in
such circuits. However, by a modification of Ohm's law which
does take into consideration the effect of reactance we
obtain a general law which is applicable to ac circuits.
Because the impedance, Z, represents the combined
opposition of all the Reactance and Resistances, this
general law for ac is,
For example, suppose a series
circuit contains an inductor
having 5 ohms resistance
and
25
ohms
inductive
reactance ( XL ) in series
with a capacitor having 15
ohms capacitive reactance (
XC ). If the voltage is 50 volts,
what is the current?

275

276

In a DC circuit the
is equal
the voltage times the
power
POWER
IN to
AC
current, or
P = E X I. Same CIRCUITS
in an AC circuit when the current and voltage
are in phase
When the current is out of phase with the voltage
Apparent Power indicated by the applied voltage and the
total current
True Power depends upon the phase angle between the
current and
voltage. The symbol for phase angle is (Theta).
Reactive Power is power that is returned to the source by
the reactive
components in the circuit .
The Apparent Power is reduced by the power returned to
the source, so that in such a circuit the Net Power, or True
Power, is always less than the Apparent power

277

Instantaneous power when current and


voltage are out of phase

Relationship between the voltage, the current, and the power


in such a circuit. The part of the power curve which is shown
below the horizontal reference line is the result of multiplying a
positive instantaneous value of current by a negative
instantaneous value of the voltage, or vice versa. Product
obtained by multiplying a positive value by a negative value
will be negative. The power at that instant must be considered
as negative power. In other words, during this time the
reactance was returning power to the source.
278

The instantaneous power in the circuit is equal to the


product of the applied voltage and current through the
circuit
In a purely resistive circuit all of the power is consumed
and none is
returned to the source .
In a purely reactive circuit no power is consumed and all
of the
power is returned to the source.
In AC circuit which contains both resistance and
reactance there
must be some power dissipated in the resistance as well
as some
returned to the source by the reactance.

279

The power in the circuit


is equal to the product of
POWER
the
applied voltage and current through the circuit.
When the voltage and current are of the same
polarity they
are acting together and taking power from the
source.
When the polarities are unlike they are acting in
opposition and power is being returned to the
source.
In an ac circuit which contains reactance as well
as
resistance, the apparent power is reduced by
the power
returned to the source.
In a circuit the net power, or true power, is
always less than
the apparent power
280

Calculating True Power in AC


Circuits
The true power of a circuit is the power actually used in the
circuit. This power, measured in watts, is the power associated
with the total resistance in the circuit. To calculate true
power, the voltage and current associated with the resistance
must be used. Since the voltage drop across the resistance is
equal to the resistance multiplied by the current through the
resistance, true power can be calculated by the formula

281

Find the true power of the circuit

Since the current in a series


circuit is the same in all parts of
the circuit

282

Calculating Reactive Power in AC


The reactive power is the power returned to the source
Circuits
by the reactive
components of the circuit.
This type of power is measured in Volt-Amperes-Reactive
( VAR )
Reactive power is calculated by using the voltage and
current
associated with the circuit reactance.
Since the voltage of the reactance is equal to the reactance
multiplied by the reactive current, reactive power can be
calculated by the formula

283

Another way to calculate reactive power is to calculate the


inductive power and capacitive power and subtract the
smaller from the larger

Since this is a series circuit, current (I)


is the same in all parts of the circuit

284

Calculating Apparent Power in AC


Circuits
Apparent power is the power that appears to the source
because of
the circuit impedance.
Since the impedance is the total opposition to ac, the
apparent
power is that power the voltage source
Apparent power is the combination of true power and
reactive power
Apparent power is not found by simply adding true
power and
reactive power just as impedance is not found by
adding resistance
and reactance.
To calculate apparent power, you may use either of the
following formulas:

285

Find the apparent power for the circuit


Recall that current in a series circuit is the same in all parts
of the circuit

286

Power
Factor
The Power Factor is a number (represented as a decimal or a
percentage) that represents the portion of the apparent
power dissipated in a circuit.
The easiest way to find the power factor is to find the cosine
of the phase angle q. The cosine of the phase angle is equal
to the power factor. You do not need to use trigonometry to
find the power factor. Since the power dissipated in a circuit is
true power, then:

If true power and apparent power are known you can use the formula shown above.

287

Another formula for power factor can be developed. By


substituting the equations for true power and apparent
power in the formula for power factor, you get

Since current in a series circuit is the same in all parts of the circuit, IR
equals IZ. Therefore, in a series circuit. To compute the power factor for the
series circuit
Another method:

Decimal number .6 could also be


expressed as 60%.

288

Power Factor Correction


In reality, only true power is consumed. The ideal situation
would be for apparent power and true power to be equal. If
this were the case the power factor would be 1 (unity) or 100
percent. There are two ways in which this condition can exist.
(1) If the circuit is purely resistive or (2) if the circuit
"appears" purely resistive to the source. To make the circuit
appear purely resistive there must be no reactance. To have no
reactance in the circuit, the Inductive Reactance (XL) and
Capacitive Reactance (XC) must be equal

Correcting the Power Factor" refers to reducing the


reactance in a circuit. This is accomplished by adding
Capacitive Reactance to a circuit which is Inductive and
Inductive Reactance to a circuit which is Capacitive
289

PARALLEL RLC CIRCUITS


There is one major difference between a Series circuit and a
Parallel circuit that must be considered. The difference is that
Current is the same in all parts of a series circuit, whereas
Voltage is the same across all branches of a parallel circuit.
Because of this difference, the total impedance of a parallel
circuit must be computed on the basis of the Current in the
Use the following formulas instead:
circuit
Series RLC Circuit

Parallel RLC Circuit

290

Find the Value of Z in the Circuit

The first step in solving for Z is to calculate the individual


branch currents

291

Using the values for IR, IL, and IC, solve for IX and IZ

Using this value of IZ, solve for Z

292

If the value for E were not given and you were asked to solve
for Z, any value of E could be assumed. If, in the example
problem above, you assume a value of 50 volts for E, the solution
would be:
First solve for the values of current

Solve for IX and IZ


Solve for Z

293

To find True power


To find Reactive power

To find Apparent power

294

295

Transformer Theory

A Transformer is a device that transfers electrical energy


from one circuit to another by electromagnetic induction
(transformer action). The electrical energy is always
transferred without a change in frequency, but may
changes in magnitudes of Voltage and Current. Because a
transformer works on the principle of electromagnetic
induction. Transformer using an AC Voltage as the input
source, the voltage and current levels can be increased or
decreased by means of a transformer

296

Transformer Theory ( Cont . )


When an alternating voltage is applied to (across) one
coil, the varying magnetic field set up around that coil
creates an alternating voltage in the other coil by
mutual induction. A transformer can also be used with
Pulsating DC, but a Pure DC. voltage cannot be used,
since only a varying voltage creates the varying
magnetic field which is the basis of the mutual induction
process.
There are two classes of transformers:
(1)Voltage transformers used for stepping up or
stepping down voltages,
(2) Current transformers used in instrument circuits
297

BASIC OPERATION OF A
In its most
basic form a transformer consists of
TRANSFORMER
A primary coil or winding ,which receives energy from the
ac source
A secondary coil or winding, which receives energy from the
primary
winding and delivers it to the load.
A core that supports the coils or windings , which
provides a path for
the magnetic lines of flux

298

Basic transformer action


The primary winding is connected to a 60 hertz ac voltage
source. The magnetic field (flux) builds up (expands) and
collapses (contracts) about the primary winding. The
expanding and contracting magnetic field around the primary
winding cuts the secondary winding and induces an
alternating voltage into the winding. This voltage causes
alternating current to flow through the load. The voltage may
be stepped up or down depending on the design of the
primary and secondary windings.

299

CORE
CHARACTERISTICS
The composition
of a transformer
core depends on such
factors as voltage, current, and frequency. Commonly
used core materials are air, soft iron, and steel.
Generally, air-core transformers are used when the
voltage source
has a high frequency (above 20 kHz).
Iron-core transformers are usually used when the source
frequency is low (below 20 kHz).

A soft-iron-core transformer is very useful where the


transformer must be physically small. The iron-core
transformer provides better power transfer than does the
air-core transformer. A transformer whose core is
constructed of laminated sheets of steel dissipates heat
readily thus it provides for the efficient transfer of power.
The purpose of the laminations is to reduce certain losses .
An important point to remember is that the most efficient
transformer core is one that offers the best path for the
most lines of flux with the least loss in magnetic and
electrical energy.

300

Core Type
Transformers
There are two main shapes of cores used in laminated-steelcore transformers. One is the Core Type, so named because the
core is shaped with a hollow square through the center. That
the core is made up of many laminations of steel

Hollow-core construction

Windings wrapped around laminations


301

Shell-Core

Transformers
The most popular and efficient transformer core is the
SHELL CORE, As shown, each layer of the core consists of
E- and I-shaped sections of metal. These sections are butted
together to form the laminations. The laminations are
insulated from each other and then pressed together to
form the core

302

TRANSFORMER
WINDINGS

The transformer consists of two coils called Winding which are


wrapped around a core. The winding that is connected to the
source is called the PRIMARY WINDING. The winding that is
connected to the load is called the SECONDARY WINDING.
Shows an exploded view of a shell-type transformer. The primary is
wound in layers directly on a rectangular cardboard form

303

Cutaway view of shell-type core with


windings

When the primary winding is completely wound, it is


wrapped in insulating paper or cloth. The secondary
winding is then wound on top of the primary winding. After
the secondary winding is complete, it too is covered with
insulating paper. Next, the E and I sections of the iron core
are inserted into and around the windings

304

Schematic Symbol For


Transformer
Typical schematic symbols for transformers. The symbol for an
air-core transformer is shown in figure (A). Parts (B) and (C)
show iron-core transformers. The bars between the coils are
used to indicate an iron core. These additional connections are
called TAPS. When a tap is connected to the center of the
winding, it is called a CENTER TAP. Figure (C) shows the
schematic representation of a center-tapped iron core
transformer

305

HOW A TRANSFORMER
WORKS

NO-LOAD
CONDITION
Transformer is capable of supplying voltages which are usually
higher or lower than the source voltage. This is accomplished
through mutual induction, which takes place when the
changing magnetic field produced by the primary voltage
cuts the secondary winding. A no-load condition is said to exist
when a voltage is applied to the primary, but no load is connected
to the secondary . Because of the open switch, there is no
current flowing in the secondary winding.

306

With the switch open and an ac voltage applied to the


primary, there is, a very small amount of current called
EXCITING
EXCITING CURRENT
flowing inCURRENT
the primary. Essentially,
what the exciting current does is "excite" the coil of the primary
to create a magnetic field.

The amount of exciting current is determined by three


factors:
1. The amount of voltage applied (Ea),
2.The resistance (R) of the primary coil's wire and core
losses
3.The XL which is dependent on the frequency of the exciting
current.

These last two factors are controlled by transformer


design.
1)Most of the exciting energy is used to maintain the magnetic
field of the primary.
2)A small amount of energy is used to overcome the resistance
Exciting
current
willlosses
flow in
the primary
winding
at all
times
of
the wire
and core
which
are dissipated
in the
form
of
to maintain
this magnetic field, but no transfer of energy will
heat
(power loss).
take place as long as the secondary circuit is open
307

Producing a Counter
EMF
When an alternating current flows through a primary
winding, a magnetic field is established around the
winding. As the lines of flux expand outward, relative
motion is present, and a counter emf is induced in the
winding. Flux leaves the primary at the north pole and
enters the primary at the south pole. The counter
emf induced in the primary has a polarity that
opposes the applied voltage, thus opposing the flow
of current in the primary. It is the counter emf that
limits exciting current to a very low value

308

Inducing a Voltage in the Secondary Winding


As the exciting current flows through the primary,
magnetic lines of force are generated. During the
time current is increasing in the primary, magnetic
lines of force expand outward from the primary and
cut the secondary. Voltage is induced into a coil
when magnetic lines cut across it. Therefore, the
voltage across the primary causes a voltage to be
induced across the secondary.

309

Primary and Secondary Phase


Relation Ship

The secondary voltage of a simple transformer may be


either in phase or out of phase with the primary voltage.
This depends on the direction in which the windings are
wound and the arrangement of the connections to the
external circuit (load). Simply, this means that the two
voltages may rise and fall together or one may rise while
the other is falling. Transformers in which the secondary
voltage is in phase with the primary are referred to as Like
Wound transformers, while those in which the voltages are
180 degrees out of phase are called Unlike Wound
transformers . Dots are used to indicate points on a
transformer schematic symbol that have the same
instantaneous polarity ( points that are in phase ).

310

Instantaneous polarity depends


on direction of winding
Same Polarity
Both the Primary and Secondary windings are wound from
top to bottom in a clockwise direction, as viewed from
above the windings. The top lead of the primary and the
top lead of the secondary have the Same Polarity. This is
indicated by the dots on the transformer symbol. A lack of
phasing dots indicates a reversal of polarity

311

Opposite Polarities
The Primary is wound in a clockwise direction from top to
bottom, while the Secondary is wound in a counterclockwise
direction. The top leads of the primary and secondary have
Opposite Polarities. This is indicated by the dots being
placed on opposite ends of the transformer symbol. Thus, the
polarity of the voltage at the terminals of the secondary of a
transformer depends on the direction in which the secondary
is wound with respect to the primary

312

Coefficient of
Coupling
Coefficient of Coupling of a transformer is dependent on the
portion of the total flux lines that cuts both primary and
secondary windings. All the lines of the flux generated by the
secondary should cut the primary.
The coefficient of coupling would be one (unity), and
maximum energy would be transferred from the primary to
the secondary. Practical power transformers use highpermeability silicon steel cores and close spacing between
the windings to provide a high coefficient of coupling

313

LEAKAGE FLUX
Lines of flux generated by one winding which do not link
with the other winding are called LEAKAGE FLUX. Since
leakage flux generated by the primary does not cut the
secondary, it cannot induce a voltage into the secondary.
The voltage induced into the secondary is therefore less than
it would be if the leakage flux did not exist. Since the effect of
leakage flux is to lower the voltage induced into the
secondary, the effect can be duplicated by assuming an
inductor to be connected in series with the primary

LEAKAGE INDUCTANCE
Is assumed to drop part of the applied voltage, leaving less
voltage across the primary.

314

Turns and Voltage

ratios
The total voltage induced into the secondary winding of a
transformer is determined mainly by the RATIO of the number
of turns in the primary to the number of turns in the
secondary, and by the amount of voltage applied to the
primary. Refer to figure A shows a transformer whose primary
consists of ten turns of wire and secondary consists of a
single turn of wire. lines of flux generated by the primary
expand and collapse, they cut BOTH the ten turns of the
primary and the single turn of the secondary. Since the
length of the wire in the secondary is approximately the
same as the length of the wire in each turn in the primary

315

Output Voltage
When the turns ratio and the input voltage are known,
the output voltage can be determined as follows:

Notice the equation shows that the ratio of secondary


voltage to primary voltage is equal to the ratio of
secondary turns to primary turns. The equation can be
written as

316

If any three of the quantities in the above formulas are known,


the fourth quantity can be calculated.
Example.
A transformer has 200 turns in the primary, 50 turns in the
secondary, and 120 volts applied to the primary (Ep). What is
the voltage across the secondary (E s)?

317

Example.
There are 400 turns of wire in an iron-core coil. If this coil is to
be used as the primary of a transformer, how many turns must
be wound on the coil to form the secondary winding of the
transformer to have a secondary voltage of one volt if the
primary voltage is five volts?

318

Das könnte Ihnen auch gefallen